Välkommen till Resurscentrums frågelåda!

 

Vill du ha ett snabbt svar - sök i databasen: Anpassad Google-sökning
(tips för sökningen).
Använd diskussionsforum om du vill diskutera något.
Senaste frågorna. Veckans fråga.

64 frågor/svar hittade

Elektricitet-Magnetism [20895]

Fråga:
När i vardagen kan man använda sina kunskaper om hur en ringklocka fungerar? Finns det närliggande områden också som man lättare kan förstå om man vet hur en ringklocka fungerar? tacksam för svar snart.
/Josefina B, Hällebergsskolan, Ljungskile

Svar:
Josefina! Kunskap är alltid bra även om den inte alltid är direkt användbar.

Om du behöver reparera en ringklocka är det naturligtvis en fördel att veta hur den fungerar.

En ringklocka kan vara konstruerad på olika sätt, se länken nedan, men klassiska ringklockor har normalt en elektromagnet. En klassisk ringklocka är därför ett bra exempel på hur en Elektromagnet fungerar. En fördel med ringklockan som exempel på en enkel elektrisk krets är att den är ofarlig eftersom den ofta drivs av batterier med mindre än 20 V spänning.
/Peter E

Nyckelord: ringklocka [5]; *vardagsfysik [64];

*

Kraft-Rörelse [20872]

Fråga:
Hej

Att kolla om ägget är gammalt har man hört och testat genom att lägga det i vatten och om det flyter så är det gammalt medan om det sjunker är det ok. Vad är det som händer med äggets densitet då det blir gammalt oh varför? Mvh Jennie
/Jennie A, Grevåker, Hammerdal

Svar:
Hej Jennie! Du har helt rätt i att det har att göra med densiteten hos ägget. Färska ägg sjunker för att de har högre densitet, gamla ägg flyter för att de har lägre densitet (Arkimedes princip). Frågan kan alltså reduceras till: varför minskar medeldensiteten hos ägg med tiden?

Det kan knappast bero på att storleken ändras - äggskalet är ganska robust och tillåter knappast storleksändring. För att minska densiteten hos vitan/gulan måste någonting med hög densiet lämna ägget.

Förruttnelseprocessen (orsakas av bakterier som alltid finns närvarande) börjar direkt. Från början är processen långsam och förruttnelsen märks inte. Vad som händer är att den flytande gulan/vitan omvandlas till gaser, bland annat CO2 och H2S. (Den senare luktar förfärligt och är en bra signal på att ett ägg är ruttet.)

Äggskalet har små porer (kycklingen behöver kunna andas), och gaserna kan smita ut genom dessa. Det inre trycket hålls konstant genom att luft kan passera genom porerna i skalet. De luftfickor som bildas har mycket lägre densitet än gulan/vitan, så äggets medeldensitet minskar med tiden.

Se även länk 1. I länk 2 (Comments from Readers) finns en, såvitt jag förstår, korrekt förklaring i överensstämmelse med förklaringen ovan.
/Peter E

Nyckelord: Arkimedes princip [32]; *vardagsfysik [64];

1 http://www.svenskaagg.se/?p=19894
2 https://whatscookingamerica.net/Eggs/EggsFloat.htm

*

Värme [20760]

Fråga:
Frysning av vatten - igen
/Veckans fråga

Ursprunglig fråga:
För att lättare förklara frågan börjar jag genom att presentera två påståenden:

Antaget att en vattenkropp med volymen V vid temperaturen av 40°C kyls ner till 0°C i tiden t.

En praktiskt likadan vattenkropp med volymen V vid temperaturen av 60°C borde kylas ner till 0°C i tiden t+x där x är tiden det tar att kyla vattnet från 60°C till 40°C. Förutsatt att kylningsförhållanden är likadana.

Med hjälp av dessa påståenden kan man anta att då det varma vattnet (60°C) sätts i en frys och då det når 40°C placeras en praktiskt likadan vattenkropp med starttemperaturen av 40°C bör de även nå frysningspunkten ungefär lika fort.

Skillnaden av mängden evaporerad vätska mellan det initialt varmare och kallare vattnet kan endast förklara några få procents skillnad. Samtidigt är ventilationen tillräckligt tillfredsställande för att luften inte ska öka i temperatur till följd av en längre period av värmeöverföring.

Det innebär att i värsta fall bör skillnaden i tiden det tar att nå frysningspunkten variera med högst 5%.

Under mitt gymnasiearbete (NV) där jag undersöker validiteten av Mpemba-effekten har jag gjort det exakta jämförelsen som förklarades i början. Totalt har 26 mätningar gjorts och vartenda försök uppvisade en skillnad mellan 9,3% (som minst) och 19,8% (som mest). Nedan finns en graf med mätvärden för 14 försök med kranvatten (de resterande 12 varierar i startförhållanden och är ej med i grafen då de avviker rent naturligt).

Frågan är då hur kan man förklara dessa fenomen? De beräkningar jag har gjort på huvudsakligen evaporation, konvektion samt ojämnt temperaturfördelning och dess påverkan på värmeöverföringen är långt ifrån de värden jag åstadkom.

Tack i förhand, Mvh Bartosz M. Piorkowski
/Bartosz P, Möckelngymnasiet, Karlskoga

Svar:
Mpemba-effekten har diskuterats här tidigare, se fråga 7830 . Det finns en hel del data som tyder på att effekten är verklig, men det finns ingen bra förklaring till effekten.

Dina resultat (figuren nedan) ser övertygande ut. Jag hade dock föredragit om du ritat in datapunkterna i stället för linjerna som jag antar är något sorts "bästa anpassning". Man hade då kunna bedöma tillförlitligheten från spridningen i datapunkterna.

Du bör även förklara varför du bortser från nästan hälften av mätvärdena.

Ett annat problem är att du, såvitt jag förstår, inte mätt tills provet börjar frysa utan till det når den normala fryspunkten 0oC. Detta är inte den urspungliga Mpemba-effekten. Det du mäter är avkylning av vatten, inte vatten som övergår i is.

Det betyder också att flera av de traditionella förklaringarna (som har att göra med frysningsprocessen) faller bort, framför allt underkylning*).

Se även Mpemba_effect#Suggested_explanations för möjliga förklaringar.

Här är en utmärkt, aktuell sammanfattning av Mpemba-effekten:

___________________________

*) Underkylning innebär att en vätskas temperatur sänks under dess smältpunkt utan att den övergår i fast form. Exempelvis kan rent vatten (regndroppar och molndroppar) vara underkylt. När den underkylda vätskan träffar fast material, exempelvis marken, kan den snabbt kristalliseras och ge upphov till nedisning. (Underkylning )



/Peter E

Nyckelord: *vardagsfysik [64]; värmeöverföring/transport [46]; vatten/is [49]; Mpemba-effekten [2];

1 http://forgetomori.com/2008/skepticism/mpemba-science/
2 https://www.sciencealert.com/mpemba-effect-warm-water-cooling-faster-than-cold-water-new-explanation

*

Ljud-Ljus-Vågor [20753]

Fråga:
Smälla en uppblåst ballong med en blixt
/Veckans fråga

Ursprunglig fråga:
hej, min pappa sa att detta med om man ritar med en svart kvadrat på armen och lysa med blixt på, och att det skulle sticka bara var tro och skrock, jag testade på honom och han erkände faktiskt att det stickte till lite, helt plötsligt kom det bara upp en annan fråga i huvudet, om man ritar en svart fläck på en ballong som inte är svart, kommer det hända något då? isåfall kommer det vara samma anledning som om man gör det på kroppen? Annars är jag väldigt tacksam om du förklarar för mig, min pappa trodde inte på detta heller men han skulle åka och hämta min syrra i skolan oxh jag tror jag lyckades övertala honom om att köpa ballonger så jag kunde testa. //Julia
/Julia p, Lovisedalsskolan, Vallenrtuna

Svar:
Hej Julia!

Kvadraten på armen och en blixt finns utredd i fråga 20427 . Att effekten är verklig är helt klart även om man inte vet exakt vad som händer. Att det har att göra med snabb uppvärmning genom att ljus från blixten absorberas är fullt klart.

Jag tror inte att ballongen påverkas i ditt försök, men det enda säkra är att göra experimentet. Hoppas du kan få tag på en ballong så du kan göra experimentet!

Uppdatering från Julia: "Min mamma köpte hem ballonger och det funkade faktiskt!"

Jag får böja mig för resultatet från ditt experiment! Tack Julia! /Peter

Den högre absorptionsförmågan på det svarta området räcker tydligen för att värma upp ett område på ballongytan tillräckligt för att ballongen skall spricka.
/Peter E

Nyckelord: *vardagsfysik [64];

*

Kraft-Rörelse [20443]

Fråga:
Kast med liten vattenflaska
/Veckans fråga

Ursprunglig fråga:
Det kallas "water bottle flip challange" och har blivit populärt i vår skola. Det går ut på att få en petflaska med lite vatten i att stå upp efter att den har snurrats ett eller flera varv. En elev i årskurs 6 undrade hur det kommer sig att den kan stå upp när den snurrats ett varv men bara studsar när man släpper den rakt ner. Går det att förklara på något enkelt sätt?
/Daniel E, Fryxellska skolan, Västerås

Svar:
För det första är det inte helt lätt att utföra tricket. Det kräver en hel del övning eftersom flaskan måste landa med botten nedåt. I videon nedan finns några lyckade försök - en del så lyckade att man undrar om de inte är fejkade .

I länk 1 finns en avancerad förklaring, men jag tror man krånglat till problemet onödigt mycket. Man förklarar inte heller varför flaskan inte studsar som den gör när man släpper den rakt ner.

Jag har gjort lite egna experiment med en 1.5 liters ICA apelsinsaftflaska till en tredjedel fylld med vatten.

Det är helt korrekt att flaskan välter nästan varje gång om man bara släpper den med botten nedåt, trots att tyngdpunkten är ganska låg. Detta beror säkert på att kontakten med underlaget får flaskan och vattnet att samtidigt studsa uppåt - vattnet befinner sig ju hela tiden längst ner i flaskan. En liten avvikelse från vertikalen ger en rotation som får flaskan att välta.

När man i stället kastar flaskan så att den snurrar exakt ett varv innan den landar har en del av vattnet flyttats högre upp i flaskan. Denna landar och försöker studsa uppåt. Innan den hinner någon vart kommer vattnet att kollidera med botten på flaskan, och flaskan trycks tillbaka till underlaget och ställer sig stadigt.
/Peter E

Nyckelord: *vardagsfysik [64];

1 http://www.vox.com/2016/5/26/11785562/water-bottle-flip-physics

*

Ljud-Ljus-Vågor [20427]

Fråga:
Blixt och stöt
/Veckans fråga

Ursprunglig fråga:
En elev hade idag målat en svart kvadrat i sitt armveck med en whiteboardpenna. Han berättade att när han höll telefonens kamerablixt mot den svarta rutan och tog ett kort fick han en stöt. Detta verkar vara känt som "sharpie shock". Jag testade själv och jag fick också en stöt. Hur kan detta förklaras? Varför känns det som en stöt?
/Gustav L, Helenelundsskolan, Sollentuna

Svar:
Det är säkert uppvärmningseffekten som förklarar liknande experiment, se fråga 11883 . Den snabba expansionen hos luften nära där blixten träffar den svarta rutan kan kännas som en stöt.

Varför känns det som en stöt snarare än en värmeeffekt direkt? Detta är fysiologi, och det är vi inte experter på. Vi kan bara spekulera att värmeeffekten är liten och kortvarig så att de temperaturkänsliga sensorerna inte triggas.

Det är emellertid klart att det finns en mekanisk effekt, se fråga 11883 . Denna borde kunna triggas av den snabbt upphettade expanderande luften, det hörs ju ett tydligt knäpp.

Det finns ett antal videos på webben, och det är uppenbart att resultatet varierar för olika position på kroppen. Detta kan förklaras av olika täthet av sensorer som registrerar stötar.

Länk 1 innehåller en video och mer eller mindre korrekta funderingar om orsaken. Länk 2 lite om farligheten. Här är en lärare som gjort en trevlig systematisk undersökning av fenomenet.


/Peter E

Nyckelord: *vardagsfysik [64]; *fysiologi [13];

1 http://www.snopes.com/sharpie-shock-challenge/
2 http://24malmo.se/2016/12/14/sa-farlig-ar-nya-stot-trenden-som-sprids-pa-internet/

*

Värme [19446]

Fråga:
Kan vatten frysa vid lufttemperaturer över noll grader.
/Veckans fråga

Ursprunglig fråga:
En fråga jag fått från elev under praktik:

Ett tunt lager vatten ligger på ett fat som är isolerat mot underlaget. På natten sjunker lufttemperaturen till 1 grader och det är molnfritt. Kan vattnet på fatet frysa? Motivera ditt svar!

Hur ska jag svara på bästa sätt?
/Jack E, Lunds Universitet, Lund

Svar:
Ja, vattnet kan frysa. Det finns två effekter som kan kyla vattnet så det fryser: utstrålning och förångning.

Om det är molnfritt är det nästan ingen instrålning av värmestrålning (infrarött/mikrovågor) från himlen. Utstrålningen är emellertid vad som motsvarar 1oC. Det betyder att vi har mer utstrålning än instrålning: temperaturen hos vattnet sjunker. Se vidare fråga 7130 .

Om omgivande luftfuktigheten inte är för hög kommer en del av vattnet att förångas. Detta kräver en energi på 2260 kJ/kg (14203 ). Energi tas alltså från vattnet för att sänka temperaturen (2.1 kJ/kg.K) och bilda is (333 kJ/kg). Denna effekt används i snökanoner, se fråga 15592 .
/Peter E

Nyckelord: vatten/is [49]; temperaturstrålning [29]; *vardagsfysik [64];

*

Blandat [19389]

Fråga:
Hej! "Étt vedträ kan inte brinna" enligt ett talesätt. Vad gör att det krävs minst två för att det skall brinna bra? Finns det någon övre "optimal" gräns för antalet vedträn i en eld?
/Thomas Å, Knivsta

Svar:
För att något skall brinna krävs det bränsle, syre och värme. Temperaturen hos t.ex. ett vedträ bestäms till en stor del av balansen mellan instrålning och utstrålning. Om omgivningen är kall blir instrålningen liten, och temperaturen låg eftersom utstrålningen dominerar. Om omgivningen däremot är varm (ett annat brinnande vedträ) ökar temperaturen genom ökad instrålning.

Detta är samma effekt som gör att rutorna inte blir frostiga på en bil i en carport, se fråga 7130 .

Vad som är "optimalt" är svårt att säga eftersom även andra effekter kommer in, t.ex. tillflödet av syre.
/Peter E

Nyckelord: strålning, in-/ut- [6]; *vardagsfysik [64];

*

Ljud-Ljus-Vågor [19218]

Fråga:
Hej! Vi fick som läxa att se om vi kunde stoppa IR-strålningen från fjärrkontrollen till tv:n och sedan också se vilka material som kunde reflektera strålningen. När jag testade att sätta ett papper framför fjärrkontrollen så gick strålningen igenom och tv:n fungerade som vanligt. När jag vände på mig och pekade fjärrkontrollen i motsatt riktning och pappret framför så fungerade det också. hur kommer det sig? Borde inte ir-strålningen gå rakt igenom, som den gjorde innan? reflekterar alla material IR- strålning? tacksam för svar!
/Alicia O, Mimers Hus, Kungälv

Svar:
Alicia! Mottagaren är ganska känslig, så den reagerar på en liten del av strålningen om bara avståndet inte är för stort. En del av IR-strålningen går alltså igenom pappret och en del reflekteras.

Nej alla material reflekterar inte IR-strålning. Liksom för ljus bör ytan vara blank för effektiv reflektion, se fråga 18817 .

Du kan faktiskt se IR-strålningen med kameran i din SmartPhone. Rikta IR-sändaren mot kameralinsen och titta på displayen. Se nedanstående bild och fråga 17944 .



/Peter E

Nyckelord: ljusreflektion [18]; *vardagsfysik [64];

*

Värme [19217]

Fråga:
Hej !

Det sägs att ifall man vill undvika sommarvärmen i sovrummet så ska man blöta ner några handdukar och hänga upp det på stolar i rummet .

Varför ska man göra så ? Vad är det som händer med atomerna/molekylerna ( detta är en kemi a fråga)
/Ann J

Svar:
Ann! Nej det fungerar nog inget vidare. I bästa fall avdunstar en del vatten vilket kyler ner handdukarna och stolarna eftersom det åtgår energi för att förånga vattnet. Denna energi tas från vattnet, som blir kallare. Om rummet är ett slutet system så blir rummet snarare varmare när vattenångan kondenseras någon annanstans. Det är mer effektivt om du sätter den våta handduken över huvudet.

Se även fråga 11227 .
/Peter E

Nyckelord: *vardagsfysik [64];

*

Elektricitet-Magnetism [18851]

Fråga:
Seriekoppling av glödlampor
/Veckans fråga

Ursprunglig fråga:
Vi labbade med eleverna seriekoppling och parallellkoppling av lampor.

Enligt teorin skall strömstyrkan för seriekoppling minska till hälften om man byter från en lampa till två lampor, bli en tredjedel om man har tre lampor och så vidare..men i praktiken blir inte resultatet det! Strömtyrkan minskade nog men mindre än det teoretiska. Tänker man sig att sladdarnas resistans påverkar oxå så borde ju strömstyrkan minska mer än det teoretiska.
/Svante K, Finland

Svar:
Svante! Din teori gäller bara om resistansen hos lamporna är konstant, dvs samma när de är tända som när de är släckta. Ledningarna bör normalt ha så litet motstånd att de inte påverkar spänningen.

Om du mäter resistansen för en släckt lampa med en ohmmeter så finner du att resistansen är c:a 40 ohm. För en 40 W lampa ansluten till 230 V blir resistansen

R = U2/P = 2302/40 = 1300 ohm.

Anledningen är helt enkelt att resistansen hos glödtråden ökar med temperaturen. Ju varmare glödtråd desto högre resistans. När man tänder lampan får man under en mycket kort stund mycket hög ström. Effekten i en kall glödtråd blir

P = U2/R = 2302/40 = 1300 W

På en bråkdel av en sekund stiger temperaturen och motståndet tills ett jämviktsläge med effekten 40 W inställer sig.

Strömmen blir enligt Ohms lag

I = U/R = 230/1300 = 0.18 A.

Vad händer om man seriekopplar två lampor? Om resistansen vore densamma skulle vi få strömmen

I = U/(2R)

Men eftersom strömmen är mindre blir även uppvämningen mindre. Du får då en lägre resistans än 2R, så strömmen blir mer än hälften av värdet med en lampa. Detta är precis vad du observerade.

Lägg märke till att lampor uppför sig helt annorlunda än resistorer! Resistorer har konstant resistans eftersom man normalt inte låter strömmen bli så hög att de värms upp till höga temperaturer.
/Peter E

Nyckelord: glödlampa [23]; *vardagsfysik [64]; serie- och parallellkoppling [19]; resistans [15];

*

Blandat [18850]

Fråga:
Hej! Jag har läst fråga 11883 och 11911 om fotoner och cymbaler att det är ett termiskt fenomen. En kollega här på skolan visade att man får ett lika högt ljud om man sätter en glasskiva i mellan blixten och cymbalen. Vi hade en rätt stor glasskiva.

Hur fungerar detta? Värms luften på andra sidan glaset upp också? Helt enkelt, hur funkar detta?
/Gustaf I, Berzeliusskolan, Linköping

Svar:
Gustaf! Du refererar till fråga 11883 och 11911 .

Eftersom ljuset går obehindrat igenom glasskivan så bör det inte göra någon skillad. Det mesta ljuset träffar den sotade ytan och absorberas. Vi får alltså en uppvärmning av folien, speciellt när den är sotad.

Detta är samma effekt som beskrivs för snurran i fråga 8366 : den sotade ytan värms upp och värmen får snurran att rotera.
/Peter E

Nyckelord: *vardagsfysik [64];

*

Värme [18517]

Fråga:
Pizzatransport
/Veckans fråga

Ursprunglig fråga:
Eva köper två pizzor och transporterar hem dem i en kartong. Vilken pizza är varmast när hon kommer hem, den översta eller den nedersta?
/Sven K

Svar:
Den undre pizzan är varmast. Anledningen är att bröd-delen av pizzan har dålig värmeledningsförmåga. Brödet verkar alltså som värmeisolation. Den nedre pizzan har ett lager bröd både nedåt och uppåt, medan fyllningen på den övre pizzan är fritt exponerad för omgivningen.

En del av avkylningseffekten är att vatten förångas från den varma pizzan. Eftersom detta kräver ångbildningsvärme kyls pizzan med samma effekt som gör att du fryser när du går ut ur duschen. Brödsidan innehåller mycket lite vatten medan fyllningssidan innehåller vatten. Det kan alltså hjälpa lite att täcka den övre pizzan med något vattentätt, t.ex. aluminiumfolie.

Den optimala transportmetoden är att lägga pizzorna med fyllningen mot varandra. Båda pizzorna omges då av ett värmeisolerande och torrt skikt. Nackdelen är att det blir kladdigt att separera pizzorna. Om man har pizzor med olika fyllning kan man emellertid få nya intressanta smakupplevelser. Och, framför allt, båda pizzorna är varma!

Se även fråga 16374 .



/Peter E

Nyckelord: värmeöverföring/transport [46]; *vardagsfysik [64];

*

Kraft-Rörelse [18438]

Fråga:
Jag undrar varför en vattenstråle blir tunnare en bit nedanför kranen?
/Veckans fråga

Ursprunglig fråga:
Hej! Jag undrar varför en vattenstråle blir tunnare en bit nedanför kranen?

Tacksam för utförligt svar.
/Claudia M, Jensens Gymnasium, Stockholm

Svar:
Intressant fråga, så jag testade det genom ett litet experiment i vad som på engelska kallas "kitchen sink physics".

En möjlig förklaring är att vattnet faller fritt när det lämnar kranen. När hastigheten ökar måste därför strålens diameter minska för att vattnets volym skall bevaras. Vi använder formlerna för fallrörelse med konstant acceleration.

I figuren nedan syns att vattenstrålen definitivt bli tunnare allteftersom den rör sig nedåt.

Låt oss börja med några allmänna uttryck för likformig acceleration (konstant acceleration, t.ex. i tyngdkraftfältet), se Acceleration#Uniform_acceleration . Man brukar använda följande beteckningar för storheterna:

v sluthastighet (vid tiden t, m/s)
u begynnelsehastighet (vid tiden 0, m/s)
s sträcka (m)
a acceleration (m/s2)
t tid (s)

Acceleration definieras som (ändring i hastighet)/(tiden) dvs

a = (v-u)/t

Genom omgruppering får vi

v = u + at (1)

Medelhastigheten ges av

(u+v)/2 = s/t

vilket kan omgrupperas till

s = [(u + v)/2] t (2)

Vi använder (1) för att eliminera v från ekvation (2)

s = ut + at2/2 (3)

Slutligen använder vi (1) för att eliminera t i ekvation (2)

s = [(v+u)/2][(v-u)/a] = (v2 - u2)/2a

vilket ger uttrycket

v2 = u2 + 2as (4)

VI kommer att använda ekvation (4) för att testa "fritt fall"-hypotesen.

Vi behöver först bestämma vattenflödet F. Vi gör detta genom att mäta tiden det tar att fylla ett enliters kärl. Det tog 90 sekunder, så flödet blir:

F = 1 [l]/90 [s] = 1*10-3 [m3]/90 [s] = 1.11*10-5 m3/s

Genom att mäta diametern hos den grå adaptern (33 mm) kunde en kalibrering för mm/pixel på bilden åstadkommas (originalet är 4 gånger större än nedanstående bild). Därefter kunde alla avstånd mätas och omräknas till mm.

I tabellen nedan finns alla uppmätta och uträknade data: avstånd från startpunkten*, strålens radie, hastigheten, fritt fall hastighet och differensen i hastighet.

____________________________________________________

D (mm) r (mm) v (m/s) vB (m/s) Differens (%) 0 3.21 0.34 50 1.77 1.12 1.05 -6% 94 1.47 1.62 1.45 -10% ____________________________________________________

Bevarande av flödet ger följande samband:

A*v = p r2*v = F

dvs

v = F/(p r2)

A är tvärsnittsytan, v är hastigheten och r är strålens radie.

För positionen D = 0 får vi t.ex.

v = 1.11*10-5/(p(3.21*10-3)2) = 0.34 m/s

Med hjälp av ekvation (4) ovan kan vi få ett beräknat värde på hastigheten om hypotesen fritt fall är korrekt:

vB = sqrt(0.342 + 2g*50*10-3) = sqrt(0.116 + 0.981) = 1.05 m/s

och

vB = sqrt(1.122 + 2g*44*10-3) = sqrt(1.25 + 0.86) = 1.45 m/s

Avvikelsen mellan de uppmätta värdena på hastigheterna och de som beräknats för fritt fall är -6% och -10%. Det är nog lite för stora avvikelser för att helt kunna förklaras som tillfälliga mätfel. Det tycks alltså komma in andra effekter, t.ex. ytspänning och laminär strömning, dvs att hastigheten inte är konstant över tvärsnittsytan, se Laminar_flow . Den svenska Wikipedia artikeln Laminär_strömning är ganska intetsägande, men innehåller ett kul skämt (bildtexten till bilden med fiskar).

I länk 1 beskrivs ett liknande experiment. Man använder sig av Bernoullis ekvation, som emellertid ger exakt samma resultat som i ekvation (4). Länk 2 behandlar acceleration och fritt fall (v-t diagram).

____________________________________________________________________

* För att undvika lokala effekter från kranen valdes nollpunkten en bit ner. Nedre punkten valdes lite ovanför 100 mm nivån eftersom bilden av strålen blir ganska otydlig.



/Peter E

Nyckelord: fallrörelse [31]; *vardagsfysik [64]; acceleration [6];

1 http://www.chabotcollege.edu/faculty/shildreth/physics/BernoulliLab.htm
2 http://www.lightandmatter.com/html_books/lm/ch03/ch03.html

*

Värme [18257]

Fråga:
Hur fungerar en värmepump?
/Veckans fråga

Ursprunglig fråga:
Hej! Kan du hjälpa mig med denna frågan? Känna till principen (mycket enkelt) för hur ett kylskåp och en värmepump fungerar
/Derya B, Komvux, Kungälv

Svar:
En värmepump är en teknisk anordning som överför värme från en kall till en varm plats. För att detta ska vara möjligt måste energi i någon form tillföras, enligt termodynamikens andra huvudsats. Tekniken i en värmepump är i princip densamma som i en kylanläggning. Den huvudsakliga skillnaden mellan dessa två är användningsområdet; värmepumpar används för uppvärmning, medan kylanläggningar används för kylning. (Värmepump )

En värmepump och ett kylskåp är alltså i princip samma sak. Enda skillnaden är att för kylskåpet är det den kalla delen som är av intresse och för värmepumpen är det den varma delen.

Enkelt uttryckt skapar man den kalla delen genom att låta en gas expandera eller en vätska förångas. Dessa processer kostar energi, så mediet kyls ner. Sedan kondenseras mediet i den varma delen med en kompressor (pump). Köldmediet (propan, ammoniak eller tidigare freoner) genomgår alltså i en kretsprocess och skapar en kall och en varm del. Energi tillförs systemet med en (vanligen elektrisk) motor.

Principen framgår av nedanstående figur från Heat_pump (New version of the figure with the arrow in the evaporator coil reversed to show correctly the heat transfer from outside environment to the inside fluid).

1 Kondensor (varm)
2 Strypanordning
3 Förångare (kall)
4 Kompressor

Vi har två sidor, den varma (1) och den kalla (3). För ett kylskåp är den kalla delen inne i skåpet och den varma delen utanför (kopparrör bakom skåpet). För en luftvärmepump är den kalla delen ute och den varma delen inne.

Kompressorn drivs av en elektrisk motor och rörsystemet innehåller ett gasformigt/flytande kylmedium som cirkulerar i röret.

Kompressorn (4) komprimerar gasen varvid temperaturen stiger (samma effekt som gör att en cykelpump blir varm). När gasen kondenserar frigörs ytterligare energi och temperaturen stiger ytterligare.

Strypanordningen (2) låter kylmedlet expandera, varvid temperaturen sjunker. Temperaturen sjunker ytterligare för att det går åt energi när kylmedlet övergår till gasform.

Man "pumpar" alltså värme från den kalla delen till den varma. Detta kan inte ske spontant eftersom termodynamikens andra huvudsats (Termodynamikens_huvudsatser ) säger att värme går från en varmare kropp till en kallare om man inte tillför arbete. Det är den elektriska motorn som genom mekaniskt arbete driver kompressorn som får värmen att gå "åt fel håll".

När det är varmt kan man med en vanlig värmepump åstadkomma kylning genom att helt enkelt pumpa kylmedlet i andra riktningen så att 1 (inne) blir kallt och 3 (ute) blir varmt.

För att sprida värmen/kylan effektivt har man en fläkt både inne och ute.

Se även fråga 14245 , Värmepump och Heat_pump .



/Peter E

Nyckelord: värmepump/kylskåp [8]; *vardagsfysik [64];

*

Värme [18250]

Fråga:
Varför har nyhuggen ved sämre eldningsvärde än torr ved?
/Veckans fråga

Ursprunglig fråga:
Varför har nyhuggen ved sämre eldningsvärde än torr ved?
/Per O, Borgarskolan, Malmö

Svar:
Nyhuggen ved innehåller mycket mer vatten än torkad ved. Det traditionella svaret är att vattnets ångbildningsvärme är stor, och att alltså mycket energi förloras till att förånga vatten. Låt oss göra en kvantitativ uppskattning.

Energiinnehållet i torr ved är enligt Energy_density#Common_energy_densities 16 MJ/kg. Ångbildningsvärmet för vatten är 2.3 MJ/kg. Om vatteninnehållet är 30% så utvecklas 0.7*16=11.2 MJ. För att förånga vatteninnehållet går det åt 0.3*2.3=0.7 MJ. Nettoenergiinnehållet blir alltså 11.2-0.7=10.5 MJ. Förhållandet torr ved/fuktig ved blir alltså 16/10.5=1.5.

Det mesta av effektförlusten kommer sig av att vattnet inte ger något bidrag till energiutvecklingen. Totalt sett är det alltså inte ångbildningsvärmet som är den stora effekten utan det faktum att massa försvinner vid torkning. Den fuktiga vedklabben ger alltså nästan samma totala energiutveckling om man torkar den.

Men den som eldat med fuktig och torr ved vet att det är en enorm skillnad! Det måste alltså vara en annan effekt som dominerar, t.ex. hur effektivt värmen transporteras till rummet.

Det är självklart att torr ved brinner bättre (vid högre temperatur) än fuktig ved. Den utstrålade effekten per m2 ges av Stefan-Boltzmanns lag (Stefan–Boltzmann_law ):

P = sT4

där konstanten s=5.67*10-8 W/m2/K4 och T är den absoluta temperaturen i kelvin. Vi har alltså för strålning ett mycket häftigt beroende av temperaturen. Låt oss anta att temperaturen är 1000 K med torr ved och 750 K med fuktig. Strålningen är då 1.334=3 gånger högre för torr ved. (De uppskattade temperaturerna är från länk 1 sidan 4.)

Men vi måste självklart bevara den totala energin, hur går det ihop? Om förbränningen sker långsamt vid lägre temperatur kommer en större andel av värmen försvinna ut i skorstenen. Eftersom man knappast vill ha någon konvektion (strömning) av rökgaser ut i rummet, så är det strålningen (värmestrålning och synligt ljus) som värmer upp spisen och rummet. Effektiviteten hos spisen är alltså mycket beroende på temperaturen, som i sin tur beror av fuktinnehållet i veden.

Se även Wood_fuel . Nedanstående bild är därifrån.



/Peter E

Nyckelord: *vardagsfysik [64]; temperaturstrålning [29];

1 http://www.forestry.gov.uk/pdf/eng-woodfuel-woodasfuelguide.pdf/$FILE/eng-woodfuel-woodasfuelguide.pdf

*

Värme [17805]

Fråga:
Varför torkar inte plastglasen och andra plastsaker i diskmaskinen medan allt glas, metall och porslinsmaterial torkar hur bra som helst?
/Lars L, N/A, Lund

Svar:
Det fordras en hel del energi för att vatten skall förångas. När diskmaskinen stoppar är allt varmt och vått. Metall, glas och porslin har tillräcklig specifik värmekapacitet (förmåga att lagra värme, se fråga 15734 ) och hygglig värmeledningsförmåga för att vattnet på ytan skall förångas. Plast, däremot, har låg specifik värmekapacitet och dålig värmeledningsförmåga, så det tillgängliga värmeenergin räcker inte till för att förånga vattnet.
/Peter E

Nyckelord: *vardagsfysik [64]; specifik värmekapacitet [25];

*

Kraft-Rörelse [17507]

Fråga:
Arbete på ett löpband
/Veckans fråga

Ursprunglig fråga:
Min fru springer på ett löpband som går att luta. Jag påstår att hon i teknisk mening inte utför mer arbete när det lutar uppåt eftersom henne massa inte flyttas uppåt. Hon påstår att det går tyngre (jag har inte provat). Denna känsla av att det går tyngre har enl. mig möjligen en fysiologisk förklaring som jag inte kan beskriva.
/Bertil S, Hackås

Svar:
Tyvärr Bertil, din fru har rätt (igen )! Man kan resonera på flera sätt, till exempel: När foten är i kontakt med bandet transporteras den lite neråt. Detta måste kompenseras i nästa steg, så det är en riktig uppförsbacke. Om du tittar noga så flyttar sig tyngdpunkten på löparen upp och ner. Om bandet lutar, så måste denna rörelse vara större för att löparen skall komma rätt i nästa steg. Att springa på ett löpband är helt ekvivalent med normal löpning med undantag för luftmotståndet.

Andra, eventuellt förvånande, fakta är att man kan bli trött utan att utföra arbete (fråga 13327 ) och att utfört arbete beror på vilket inertialsystem man befinner sig i (fråga 14380 ).
/Peter E

Nyckelord: *vardagsfysik [64]; arbete [24];

*

Kraft-Rörelse [17076]

Fråga:
Hej! Det går att lyfta ganska många böcker med hjälp av bara en liten plastpåse och ett sugrör. Hur kommer det sig?
/Marie G, Önnestads skola, Önnestad

Svar:
Hej Marie! Det är analogt (liknar) effekten man får med en hävstång, dvs man förstärker en kraft genom att betala priset i sträckan kraften verkar på, se fråga 15286 .

Plaspåsen fungerar som ett hydraulik-system (se nedanstående figur från Wikimedia Commons, Hydraulik ). Om du trycker på den vänstra cylindern får du ett övertryck i vätskan (ofta någon sorts olja). Detta övertryck transporteras av vätskan till nedre ytan av den högra cylindern. Eftersom totala kraften F är trycket*ytan=p*A får vi

p = F1/A1 = F2/A2

och alltså F2 = F1*(A2/A1). Eftersom A2 är större än A1 får vi en förstärkning av kraften F1.

Hydraulik-system används vanligen i bromsar för personbilar.

Plastpåsen med sugröret fungerar precis på samma sätt men det kallas pneumatik i stället för hydraulik eftersom vi har en gas i stället för en vätska.

Pneumatik-system används i bromsar för lastbilar och tåg eftersom det skulle vara risk för överhettning av bromsoljan. (Du har säker hört pysandet när övertrycket släpps ut.)

En domkraft man använder för att lyfta bilen är antingen en sorts hävstång eller ett hydraulik-system.



/Peter E

Nyckelord: hävstång [5]; *vardagsfysik [64];

*

Ljud-Ljus-Vågor [16880]

Fråga:
Test av mikrovågsugn
/Veckans fråga

Ursprunglig fråga:
Jag har fått höra att man i framförallt USA testar mikrovågsugnar regelbundet (någon gång per år) för att se om de börjat läcka mikrovågor. Detta görs hemma genom att man kan hålla ett lysrör i närheten av mikron, lyser det så är ugnen för gammal och ska kasseras. Ligger någon som helst sanning i detta, att mikron kan bli gammal och börja läcka?
/Annika L, Lund

Svar:
Annika! Jag tror du blivit utsatt för ett skämt! Om det finns så mycket mikrovågor utanför ugnen att ett lysrör tänds, så är det mycket illa!

Mikovågorna hålls innne i ugnen genom att den är konstruerad som en faradaybur (se fråga 8879 ) - solid metallplåt inne i ugnen och ett finmaskigt metallnät på luckan. Maskorna är betydligt mindre än mikrovågornas våglängd, så dessa kan inte slippa ut. Enda möjligheten är att dörren eller nätet skadats, men det kan man lätt se. Man behöver alltså inte testa om ungen läcker mikrovågor. Om man vill mäta strålningen (t.ex. för att testa en ny konstruktion) bör man ha mycket bättre mätapparatur än ett lysrör.

Se fråga 16041 om vad som händer med mikrovågsugnar när de blir gamla. Se mer om mikrovågsugnar: mikrovågsugn , länk 1 från strålsäkerhetsmyndigheten och länk 2.

Se fråga 3969 om hur effektiv faradayburen i en mikrovågsugn är.

Hur man kontrollerar effekten hos en mikrovågsugn

När mikrovågsugnen blir gammal blir magnetronen som genererar mikrovågorna mindre effektiv, och det tar längre tid att värma maten. Så här kan du kontrollera hur effektiv din mikrovågsugn är. Du behöver bara en skål (glas eller keramik) med 1 liter kallt vatten och en hyggligt exakt termometer.

Mät temperaturen på vattnet. Säg att vattnet är 20oC. Kör ugnen 1 minut. Mät vattentemperaturen igen. Säg att vattnet nu är 30oC. Sedan får vi räkna lite för att få fram effekten. Energin som krävs för att värma vattnet DT K är:

W = m*c*DT

där m är massan och c är vattnets specifika värmekapacitet 4180 J/(kg K).

Om vi kör mikron under tiden t får vi effekten

P = W/t = m*c*DT/t

1 liter vatten väger 1 kg, så effekten blir

P = 4180*DT/t

Om vi körde mikron 1 minut och temperaturskillnaden var 10 K (eftersom vi har att göra med temperaturskillnader kan vi använda Celsius eller Kelvin) blir effekten

P = 4180*10/60 = 697 W

Om temperaturdifferensen blir liten bör man öka tiden t för att få bättre noggrannhet.

Den uppmätta effekten jämförs sedan med den nominella effekten enligt bruksanvisningen. Man kan även mäta in-effekten direkt med en wattmeter (se Watt_meter , bilden nedan). Om effekten är betydligt lägre än vad den skall vara är mikrovågsugnen skadad.

Om ovanstående är för krångligt kan man om man har en "standardportion", t.ex. ett fruset halvt franskbröd, helt enkelt se om upptiningen tar längre tid än vanligt.



/Peter E

Nyckelord: mikrovågsugn [25]; specifik värmekapacitet [25]; *vardagsfysik [64];

1 http://www.stralsakerhetsmyndigheten.se/Allmanhet/Magnetfalt--tradlos-teknik/Mikrovagor/Mikrovagsugn/
2 http://www.pat-testing.info/microwave-safety.htm

*

Kraft-Rörelse [16802]

Fråga:
Hej, jag undra en sak inom fysik. Om man åker i en hiss arbetar man då?
/lessi o, tyska, gbg

Svar:
Lessi! Väldigt lite arbete: bara när du trycker på knappen! Arbete är ju kraften*(vägen i kraftens riktning), se fråga 13327 , och det är inte du utan hissmotorn som utför arbetet.

Dessutom har en kabelhiss (det finns även andra typer) en motvikt som väger lite mer än den tomma hissen, se länk 1 och 2. Motvikten är till för att minimera arbetet som krävs att köra hissen upp och ner. Eftersom en typiskt lastad hiss väger ungefär lika mycket som motvikten, så behöver motorn i stort sett bara övervinna friktionen, resten av abetet utförs av motvikten.

Se även Hiss och nördversionen Elevator .
/Peter E

Nyckelord: arbete [24]; *vardagsfysik [64];

1 http://science.howstuffworks.com/elevator3.htm
2 http://www.explainthatstuff.com/how-elevators-work.html

*

Kraft-Rörelse [16763]

Fråga:
Jag har noterat att det kan gå vågor i en kaffekopp eller i toaletten inne i huset när det blåser ordentligt ute, detta trots att man inte kan känna några vibrationer i själva huset.

Är det vinden som på något sätt ger rörelseenergi till huset som sedan "löser ut" i vätskan?

Finns det någon teknisk term för vad man kallar ett sådant fenomen?
/Martin Ö, Tullinge

Svar:
Kaffekoppen är nog små vibrationer i huset. De kan vara mycket små så de inte märks men förstärkas med resonans i kaffekoppen. Varför skulle huset ha just den frekvens som är resonansfrekvens till kaffekoppen hör jag den klentrogne säga? Ja, huset vibrerar säkert med många frekvenser - vad man kallar ett vitt spektrum - eftersom vindpåverkan är mycket slumpmässig med turbulens mm. De frekvenser som passar i kaffekoppen kommer att förstärkas av resonans.

Vad gäller toalettstolen är det nog en annan effekt. Man kan se att vattennivån i toalettstolen varierar med någon cm när det blåser mycket ute. Det beror på att lufttrycket inne varierar lite grann på grund av luftströmmar i ventilationen. En variation på bara en tusendel i lufttrycket ger upphov till en cm:s skillnad i vattennivån.

Jag vet ingen annan teknisk term än resonans och lufttrycksvariationer för dessa fenomen.
/Peter E

Nyckelord: ljud, resonans [19]; *vardagsfysik [64];

*

Värme [16521]

Fråga:
Varför känns utandningsluften varmare om man andas ut med vidöppen mun jämfört med mer stängd mun?
/Veckans fråga

Ursprunglig fråga:
En av mina elever undrade varför utandningsluften känns varmare om man andas ut med vidöppen mun jämfört med mer stängd mun. Jag har själv provat och det stämmer nog men varför? Beror det på om luften kommer nere från lungorna eller från munnen?
/Marianne L, Frostaskolan, Hörby

Svar:
Marianne! Nej, luften kommer i båda fallen både från munnen och lungorna. Skillnaden är att om du stänger munnen och bara lämnar ett litet hål, så skapas ett litet övertryck i munnen. Eftersom det är ett lite lägre tryck utanför munnen, måste den utströmmande luften expandera. Om en gas får expandera fritt så kyls den. Tvärtom, om en gas komprimeras så värms den upp. Du har säkert märkt att cykelpumpen blir varm när du pumpar upp däcket.
/Peter E

Nyckelord: *vardagsfysik [64];

*

Elektricitet-Magnetism [16427]

Fråga:
Vi har en 4 år gammal mikrovågsugn. Min lillebror Zakarias sätter ibland på mikron när den är tom. Då kommer det blixtar i mikron. Varför då? Är det farligt när det blixtrar? Betyder det att mikron är trasig?
/Axel L, Enskilda, Uppsala

Svar:
Axel! Om man kör en tom mikro har energin i mikrovågorna ingenstans att ta vägen. Till slut kommer ändå vågorna att absorberas i metallen i väggarna. Då kan det byggas upp så höga spänningar att man får överslag (gnistor). Detta är definitivt inte bra, och magnetronen (apparaten som gör mikrovågorna) kommer snabbt att förstöras. Så om din lillebror inte slutar med ofoget kommer han att få dricka kall välling!

Se mikrovågsugn för andra aspekter på mikrovågsugnar.
/Peter E

Nyckelord: mikrovågsugn [25]; *vardagsfysik [64];

*

Värme [16374]

Fråga:
Varför bränner man sig på fyllningen i en pizza men inte på brödet?
/Veckans fråga

Ursprunglig fråga:
Hej! Håller på med värme i skolan. I läroboken stod ett exempel om att man bränner sig i gommen på pizza därför att fyllningen kan lagra mer inre energi än brödet, men de har samma temperatur. Min lärare har sagt att saker känns olika varma och att man bränner sig på grund av att värmeöverföringen är olika för olika ämnen. Jag undrar ifall att man bränner sig beror på värmeöverföringen/konduktiviteten hos ämnet eller om det beror på specifika värmekapaciteten? Hänger detta ihop?
/Maria H

Svar:
Hej Maria! Låt oss resonera systematiskt! Vad menas med att du bränner dig på gommen? Jo det är att gommen blir så varm (hög temperatur) att de värmekänsliga cellerna signalerar hög temperatur eller i värsta fall att man får en liten (övergående) skada. Det väsentliga är alltså att höja gommens temperatur.

För att höja gommens temperatur krävs dels att pizzan har tillräcklig specifik värmekapacitivitet och dels att värmeledningsförmågan är tillräcklig (och naturligtvis att den är tillräckligt varm). Så svaret är alltså: både och! Om värmeledningsförmågan är dålig, är det bara ett tunnt skikt som bidrar: liten temperaturhöjning eftersom temperatursänkningen hos pizzan är stor. Om specifika värmekapacitiviteten är liten, får man igen en stor sänkning av pizzans temperatur.

Metaller har mycket hög värmeledningsförmåga (eftersom det finns fria elektroner) och rimligt hög specifik värmekapacitivitet (värmemängd per kg och K). Man bränner sig alltså mycket på metaller. Du bränner dig på spiken i bastun men inte på trätrallorna. Du bränner dig inte heller på en het aluminiumfolie, eftersom massan är så liten och därmed värmekapaciteten.

Vatten har mycket hög specifik värmekapacitivitet och hygglig värmeledningsförmåga. Torra bitar som brödet har emellertid låg värmekapacitivitet och dålig värmeledningsförmåga eftersom brödet innehåller en massa luftbubblor som leder värme dåligt. Det är alltså på fyllningen (som innehåller mycket vatten) och inte på brödet du bränner dig.

/*fa*



/Peter E

Nyckelord: värmemängd [3]; värmeöverföring/transport [46]; specifik värmekapacitet [25]; *vardagsfysik [64];

*

Kraft-Rörelse [16210]

Fråga:
om har två glas halv fyllda på en våg och stoppar in fingret i ena glaset vad händer (OBS!) dett är en balans våg inte en våg som man väger sig på !!
/bymnet t, kungsskolan, kungsängen

Svar:
Bymnet! Den sida du stoppar ner fingret i rör sig nedåt, dvs blir tyngre. Du kan resonera på två sätt:

1 Fingret tar ju plats i glaset, så vattenytan i glaset stiger. Då stiger även trycket på botten av glaset. Eftersom basytan är konstant, ökar kraften.

2 Enligt Arkimedes princip får man en lyftkraft på fingret motsvarande det undanträngda vattnets vikt. För att jämvikt skall råda måste fingret påverka vattnet med samma kraft nedåt. Vikten på fingersidan ökar alltså med volymen*(vattnets densitet)*g.
/Peter E

Nyckelord: Arkimedes princip [32]; *vardagsfysik [64];

*

Kraft-Rörelse [16186]

Fråga:
Om man har en bägare till hälften fylld med vatten och stoppar ner den i en vanna med vatten så, åker inte vattnet ut från bägaren, den är under vatten. Men när man tar upp bägaren upp från vannan så åker vattnet ut direkt. Varför?
/Dimer O, Stockholm

Svar:
Så länge bägaren finns i vannan hindrar vattnet att bägaren töms. Det extra trycket från vattenpelaren i bägaren kompenseras av att lufttrycket i övre delen av bägaren är lägre än utanför. Man har alltså jämvikt. När man lyfter ut bägaren kan vattnet rinna ut om inget annat hindrar det. Se även nedanstående frågor.
/Peter E

Se även fråga 16150 och fråga 952

Nyckelord: lufttryck [23]; *vardagsfysik [64];

*

Kraft-Rörelse [16178]

Fråga:
När jag och min kompis gjorde ett experiment (vi skulle göra en kompass av en magnetiserad nål på en korkbit i en stor skål vatten), upptäckte vi att korkbiten drogs till kanten av skålen. Det hade inget med nålen att göra. När vi la ner korkbiten i mitten igen och stoppade ner fingret i vattnet nära korken drogs den till fingret. Samma sak hände när vi stoppade ner en glasskiva och en plåtbit. Vi testade också med en bit trä istället för korken med samma resultat.

Varför hände detta?
/Malte R, Furulundsskolan, Halmstad

Svar:
Hej Malte! Bra att du inte bara följer instruktioner för ett experiment och konstaterar: vi fick det väntade resultatet! Du observerar något oväntat och försöker förstå detta. Du kan säkert bli en bra vetenskapsman!

Orsaken till att korken uppför sig som den gör är vattnets ytspänning, som är ett fysikaliskt fenomen som uppstår i gränsytan mellan två faser beroende på skillnaden i energi mellan molekylerna vid ytan respektive fasens inre.

I det inre av en vätska växelverkar molekylerna i alla riktningar till skillnad från molekylerna vid en gränsyta (se bilden nedan från Wikimedia Commons). Ytmolekylerna binds därför av ett färre antal molekyler. Då fler bindningar medför lägre energi har molekylerna vid ytan högre energi än de inre molekylerna i vätskan. Ytan får således ett överskott av energi, ytenergi, med dimensionen energi per ytenhet (J/m2).

Ytenergin medför att ett system strävar efter att minimera sin yta. En fri vätskedroppe antar därför sfärisk form. Ytenergin kan även tolkas som kraft per längd, en spänning, därav namnet ytspänning, med enheten N/m (J/m2=Nm/m2=N/m).

1 Man kan även se att om ytan är konvex som i nedanstående figur så har en ytmolekyl färre grannar än om ytan är plan. Vi får alltså mindre bindning och därmed högre energi ju mer ytan är krökt. Vi har alltså minimal energi där krökningsradien är minimal.

2 På motsvarande sätt gäller att om ytan är konkav så har en ytmolekyl fler grannar än om ytan är plan. Vi får alltså högre bindning och därmed lägre energi ju mer ytan är krökt. Vi har alltså minimal energi där krökningsradien är maximal.

I fallet du beskriver är skålen säkert inte helt full. Då kommer vattnet att krypa upp längs kanterna och vi har fall 2, en konkav yta. Korken vill då till området med lägst energi, dvs där den är mest krökt - vid kanten. Om du vill att korken skall stanna i mitten bör du fylla skålen lite över kanten. Du får då en konvex yta, fall 1, och lägsta energin är i mitten.

Att olika föremål som flyter på en yta (ytan kan vara helt plan) tenderar att attrahera varandra beror på att de konvexa ytorna (föremålet trycker ju ner ytan omkring sig) omkring föremålen har hög energi, och det är fördelaktigt att slå ihop dem.

Se vidare Surface_tension .



/Peter E

Se även fråga 11165

Nyckelord: ytspänning [18]; *vardagsfysik [64];

*

Elektricitet-Magnetism [16041]

Fråga:
På mitt jobb har vi ca 10 st mikrovågsugnar på ett område på ca 3m. De flesta av dessa ugnar är värdelösa. Det tar 7-8 minuter att värma maten, som ändå är tinad. Va beror detta på, vi är många som funderar på det. De flesta av ugnarna är ändå mindre än 1 år gamla..
/Malin J, Töreboda

Svar:
Hej Malin! Jag får känslan att du har hypotesen att problemet skulle vara att ugnarna står tätt ihop. Jag tror inte det. Ugnarna är konstruerade så att mycket lite mikrovågor slipper ut, så de har inte mycket kontakt med varandra.

Magnetronen som genererar mikrovågorna slits ut med tiden och kan även skadas om man kör ugnen utan last. Man skall här tänka på att mikrovågorna växelverkar med fett och flytande vatten, men mycket sämre med is. Man skall alltså först tina maten på lägre effekt och sedan värma upp den på full effekt.

Livslängden för mikrovågsugnar varierar mycket, se 'durability' i testerna under länk 1. De flesta mikrovågsugnar som säljs är avsedda för hemmabruk, dvs att köras några minuter per dag. På en stor arbetsplats körs ugnarna säkert mycket mer, och användarna är nog lite mer slarviga än när de använder sin egen mikro, så det är inte konstigt att ugnarna slits ut snabbt.

Se länk 2 och Microwave_oven för mer om mikrovågsugnar. Se fråga 16880 för information hur man mäter effekten hos en mikrovågsugn.
/Peter E

Nyckelord: mikrovågsugn [25]; *vardagsfysik [64];

1 http://www.epinions.com/reviews/pr-GE_JEM31GY_Microwave_Oven
2 http://home.howstuffworks.com/microwave.htm

*

Ljud-Ljus-Vågor [15928]

Fråga:
Varför gnistrar diamantringar vackrare i sol-, stearin-, och halogenljus än i vanlig glödlampe- eller lysrörsljus.
/Veckans fråga

Ursprunglig fråga:
Jag fick en fråga av en elev om varför diamantringar gnistrar vackrare i sol-, stearin-, och halogenljus än i vanlig glödlampe- eller lysrörsljus. Har det något med våglängder att göra?
/Louise N, Klöverbackens skola, Kungälv

Svar:
Louise! Skönhet ligger i betraktarens öga . Jag tror inte det har så mycket att göra med vilka våglängder ljuskällan sänder ut - för ögat är det inte så stor skillnad på de ljuskällor du nämner. Jag tror att det är viktigare att ljuskällan är liten (nära punktformig) och inte diffus (utbredd). Om du t.ex. har ett stearinljus som källa får du en massa spegelbilder av ljuslågan i diamanten. Det ser mycket vackrare ut är spegelbilden av t.ex. ett lysrör.

En del av ljuset kommer även till ögat efter att ha passerat genom delar av diamanten och därmed delats upp i olika färger som ett spektrum. Om ljuskällan är stor kommer spektrum att bli otydligt (de olika färgerna överlappar och ger intryck av vitt ljus). Det är därför man har en smal ingångsspalt i en spektrograf.

Se även fråga 14331 .
/Peter E

Nyckelord: *vardagsfysik [64]; #ljus [63];

*

Värme [15789]

Fråga:
Vad är det exakt som gör att ett ägg sprängs om man lägger det i mikron, och varför sprängs det med sån kraft?
/Therese N, Högskola

Svar:
De inre, halvflytande delarna värms upp av mikrovågorna. Uppvärmningen ger upphov till en expansion. Skalet expanderar mindre, så expansionen av de inre delarna bygger upp ett övertryck. När trycket är tillräckligt stort exploderar ägget, och mikron är full av gegg . Gör inte detta hemma - mamma kommer att bli riktigt arg!
/Peter E

Nyckelord: mikrovågsugn [25]; *vardagsfysik [64];

*

Värme [15731]

Fråga:
Hur kan man beskriva avdunstning av vatten vid normal rumstemperatur på molekylnivå? Om 1kg vatten avdunstar vid rumstemperatur och 1kg vatten kokas bort vid 100 grader i vilket fall förbrukas mest energi?
/Bo S

Svar:
Vad gäller avdunstning vid rumstemperatur så skall man se det så att molekylerna har en fördelning i hastighet - vissa molekyler rör sig snabbt och vissa långsamt. De snabba molekylerna har högre sannolikhet att komma loss från vätskan, så det är dessa som försvinner. Eftersom temperaturen är ett mått på medelrörelseenergin, så kommer vätskan att kylas - en effekt man märker när man går ut ur duschen (du fryser när du är våt på väg ut, men inte när du är torr på väg in). Naturligtvis kommer avdunstningen då att minska, såvida man inte värmer upp vätskan.

Enligt länk 1 är ångbildningsvämet för vatten av temperaturen 0oC c:a 10% högre än det vid 100oC. Det kostar alltså mer energi att avdunsta kallt vatten.
/Peter E

Nyckelord: vatten/is [49]; *vardagsfysik [64];

1 http://www.btinternet.com/~martin.chaplin/data.html#Enthalpy of Vaporization

*

Kraft-Rörelse [15619]

Fråga:
Att öppna glasburkar med metallock
/Veckans fråga

Ursprunglig fråga:
Jag upplever att glasburkar med metallock går lättare att öppna om man spolar varmt vatten på. Vad är det som händer? Varför blir det så? Har det med att göra att metallocket sväller av värmen fortare än vad glaset gör och på så sätt lossnar det? Eller har det nåt med vakuum att göra att det upplöses när varmt vatten spolas på?
/Malin D, Lärarhögskolan, Stockholm

Svar:
Malin! Du har bra idéer om orsaken. Framför allt två effekter kan spela in. Den första effekten om man värmer burken snabbt och den andra om man värmer den länge.

Termisk expansion (att material utvidgar sig när temperaturen blir högre): Metaller utvidgar sig mer än glas, se tabell i artikeln Coefficient_of_thermal_expansion . Lägg märke till att den öppna delen av locket expanderar som om den vore gjord av metallen. Utvidgningskoefficienten för hål är alltså lika med den för omgivningen, se länk 1 (stycket "Thermal expansion : expanding holes") för bevis. Dessutom leder metall värme bra, medan glas leder värme dåligt. Locket blir alltså varmare än glaset, vilket förhöjer effekten att locket expanderar relativt burken. Det blir då lättare att lossa locket dels för att kontakten med burken blir mindre, och dels för att eventuella tryckskillnader kan utjämnas (se nedan).

Att ett hål expanderar som omgivningen används i mekanisk industri för något som kallas krympförband (Shrink-fitting ).

Undertryck: Locket sätts på vid en förhöjd temperatur. För en ideal gas gäller den ideala gaslagen (se fråga 15294 eller Gas_laws ) att

pV = nRT

Vid konstant volym V och konstant mängd gas (n) är tryckändringen Dp proportionellt mot temperaturändringen DT. Vi får

Dp/p = DT/T

Om omgivningstemperaturen är 20oC = 293 K och temperaturen när locket sattes på 100oC får vi

Dp = p*80/293 = 1.013*105*80/293 = 28000 N/m2 (pascal, Pa)

Om locket har en radie på 3 cm är ytan 32*p, dvs c:a 30 cm2. Totala kraften på locket blir då

0.0030*28000 = 84 N. Detta motsvarar kraften som krävs för att lyfta c:a 8 kg, alltså en ganska stor kraft.

Om man värmer upp burken genom att hålla den länge under varmvattenskranen reduceras tryckskillnaden och därmed kraften. Observera alltså att tricket att hålla burken under varmvattenkranen hjälper för båda förklaringarna ovan!

Mikroskopisk förståelse av gaslagen:
Man kan förstå varför en viss mängd luft ger lägre tryck vid lägre temperatur. Temperatur är ett mått på molekylernas medelhastighet - om vi har låg temperatur så rör sig molekylerna långsamt. De kommer därför att kollidera med väggarna mindre ofta och mindre våldsamt än om temperaturen är hög. Det är just molekylernas kollektiva effekt på väggarna som makroskopiskt (i vår värld, till skillnad från molekylernas mikroskopiska värld) uppfattas som tryck.
/Peter E

Nyckelord: utvidgning [8]; gaslagen, allmänna [24]; *vardagsfysik [64];

1 http://physics.bu.edu/~duffy/py105/Temperature.html

*

Värme [15592]

Fråga:
Hur fungerar en snökanon?
/Veckans fråga

Ursprunglig fråga:
Hur fungerar en snökanon. gärna detaljerad beskrivning
/Mikael N, Ekeby skola, Ekeby

Svar:
Det finns i pricip två typer av snökanoner. En använder sig av vatten under högt tryck och en kraftig fläkt. Den andra typen har både vatten och luft under högt tryck. Principen är att munstycket utformas så att det bildas små droppar. Dropparna är såpass små att de håller sig svävande ett bra tag så att de hinner växa och frysa.

Konstgjord snö är skild från naturlig snö i det att den består av små iskristaller och bildar inte flingor som naturlig snö. Anledningen är helt enkelt att tiden för att bilda snö är mycket längre i naturen. Annars bildas snön på samma sätt: vattenmolekyler samlas på en kondensationskärna och bildar en droppe. Droppen fryser och fortsätter under sin färd genom luften att fånga in vattenmolekyler.

Det går faktiskt att göra konstgjord snö även om temperaturen ligger lite över 0oC, speciellt om luften är torr. Det finns två effekter som kan åstadkomma den avkylning som krävs.

1 Om man använder tryckluft så kommer luften att avkylas när den får expandera. Detta gäller allmänt: en gas som expanderar avkyls, en gas som komprimeras (trycks ihop) värms upp (se gaslagen, allmänna ). Det senare har du säkert märkt när du pumpat ett cykeldäck - cykelpumpen blir varm efter en stund.

2 Vatten har mycket hög s.k. ångbildningsvärme, dvs det fordras mycket energi för att förvandla vatten till vattenånga. Denna energi tas från vattnet som alltså blir kallare. Om luftfuktigheten i omgivningen är låg, kommer vi att få mycket avdunstning från vattendropparna, och därmed mycket avkylning.

Snökanonens funktion, speciellt vid temperaturer över 0oC, illustrerar alltså två fysikaliska effekter: att expanderande luft avkyls och att en vattendroppe i torr omgivning avdunstar och kyls därmed ner av ångbildningsvärmet.

Se vidare Snow_cannon och länkarna nedan. Länk 1 är en mycket detaljerad och ändå lättillgänglig genomgång av många aspekter på ämnet. Länk 2 innehåller en video som visar hur en snökanon fungerar.
/Peter E

Nyckelord: iskristaller [5]; vatten/is [49]; *vardagsfysik [64];

1 http://travel.howstuffworks.com/snow-maker.htm
2 http://www.snow-maker.com/how.html

*

Ljud-Ljus-Vågor [15570]

Fråga:
Elektromagnetisk strålning i vardagen
/Veckans fråga

Ursprunglig fråga:
Hur mycket strålning utsänder elektriska apparater, som vi använder dagligen (som mikrovågsugn, TV, hårtork). En stjärna, hur mycket strålning utsänder den? Beror det på hur stor stjärnan är eller hur långt borta den är?
/Martina J, Malmö

Svar:
Strålning kan vara mycket olika saker även om man begränsar sig till elektromagnetisk strålning. Allting som inte är vid absoluta nollpunkten sänder ut elektromagnetisk strålning, s.k. temperaturstrålning . Allt som belyses med elektromagnetisk strålning reflekterar en viss del av strålningen.

Hur mycket olika apparater strålar är svårt att säga. I vilken enhet? Watt, fotoner/sekund eller i förhållande till skadliga nivåer? Eftersom endast det senare är av praktiskt intresse koncenterar vi oss på dessa. Låt oss gå igenom det elektromagnetiska spektret med utgångspunkt från nedanstående bild (Bilden är från NASA och därmed fri att användas med angivandet av källan. Den finns i större skala på Wikipedia Electromagnetic_spectrum .)

Radiovågor

Anses vara ofarliga eftersom de innehåller mycket lite energi och dessutom går rakt igenom kroppen. Apparater med radiosändare är t.ex. trådlös telefon, trådlöst nätverk.

Mikrovågor

Dessa innehåller mer energi och är dessutom i resonans med vattenmolekylen. Detta betyder att de absorberas i kroppen och kan ge upphov till en viss uppvärmning. Faran med mobiltelefoner är mycket diskuterad, men även mikrovågsugnar läcker en del mikrovågor, se mikrovågsugn .

Infrarött (värmestrålning)

Fjärrkontroller använder ofta infrarött, men de är så svaga at de knappast utgör någon fara. Värmeelement och spisplattor strålar i infrarött, men eftersom huden stoppar stålningen samtidigt som den är känslig för värmen, så bör värmeelement normalt inte utgöra någon fara. Största delen av effekten i glödlampor strålar i infrarött, så glödlampor är egentligen bättre som värmeelement än ljuskällor.

Synligt ljus

Synligt ljus är en form av elektromagnetisk strålning med en våglängd mellan cirka 390 och 770 nanometer. Genom att ögat är känsligt för strålning i just detta intervall, kan vi se vår omgivning. (Ljus )

Ljus kommer från glödlampor, lysrör och lysdioder. Synligt ljus har mycket liten inträngningsförmåga i kroppen, varför strålningen knappast utgör en fara - bara ögat påverkas. Lysdioder (små lasrar) finns även i t.ex. CD-spelare. Ljuset från dessa kan i princip vara skadligt, men bara om man skruvar isär apparaten.

Ultraviolett

Kommer från UV-lampor. Kan orsaka brännskador vid överdriven exponering. Kan på längre sikt även orsaka hudcancer.

Solen är annars den viktigaste och starkaste källan för infrarött, synligt ljus och UV-strålning. Även stjärnor strålar mest i dessa våglängder, men eftersom effekten avtar med kvadraten på avståndet så är effekten från stjärnor helt försumbar.

Röntgenstrålning

Kommer naturligtvis från röntgenapparater, men de har man knappast hemma. Eftersom en gammal tjock-TV ritar bilden med 20 keV elektroner, så kommer det lite röntgenstrålning från bildskärmen. Platt-TV fungerar på ett annorlunda sätt, så de ger ingen röntgenstrålning.

Röntgenstrålning och gammastrålning är vad som betecknas joniserande strålning. Den är genomträngande och kan jonisera (slita loss elektroner) materia den träffar. Joniserande strålning är därför skadlig - speciellt kan den orsaka skador på DNA och därmed, på sikt, cancer.

Gammastrålning

Gammastrålning är en del av den naturliga strålningsbakgrunden från bergarter och från byggnadsmaterial. Så länge bakgrundsnivån inte är mycket förhöjd får man betrakta den som ofarlig - den går knappast att undvika.

Annan strålning

Alfastrålning är inte elektromagnetisk strålning utan består av He-kärnor som utsänds från tunga kärnor. Alfastrålning förekommer i brandvarnare (därför skall dessa när de är slut tas om hand på ett ordnat sätt). Radon , som finns i varierande mängd överallt i bostäder mm, är i vissa fall definitivt ett hälsoproblem.

Har jag glömt något? Det har jag säkert. Vi har inte heller diskuterat magnetfält och elektriska fält som bildas av elektriska apparater. Skadligheten hos dessa är dåligt känd och ganska kontroversiell.

Figuren nedan innehåller mycket nyttig information om elektromagnetisk strålning. Överst visas t.ex. att endast radiostrålning och synligt ljus släpps igenom jordens atmosfär. Termometern längst ner visar vilka våglängder som utstrålas vid olika temperaturer. Synligt ljus utsänds alltså vid temperaturer mellan 1000 K (c:a 700oC) och 10000 K.



/Peter E

Nyckelord: elektromagnetisk strålning [21]; strålning, faror med [26]; *vardagsfysik [64]; mobiltelefon, strålning från [6];

*

Kraft-Rörelse [15515]

Fråga:
Varför kyls ett spänt gummiband när spänningen upphör?
/Veckans fråga

Ursprunglig fråga:
Hej Om jag håller en tom ballong mot min läpp t.ex., känner jag en viss temperatur. Töjer jag sedan i ballongen och håller den mot läppen känner jag en högre temperatur. Låter jag sedan ballongen återgå till sin ursprungliga form känner jag direkt den ursprungliga temperaturen. Vad har skett?
/Ragnar J, Lärcentrum, Tanumshede

Svar:
Det går bra med ett gummiband också!

Gummi har ovanliga egenskaper och uppför sig så, se Natural_rubber#Elasticity . Uttänjning av gummi utvecklar värme och kontraktion kräver värme. Teoretiskt är det analogt med adiabatisk (utan värmeutbyte med omgivningen) expansion/kontraktion av en ideal gas - bara tvärt om (gasen värms vid kompression och kyls vid expansion).

För "vanliga" elastiska material (t.ex. metaller) lagras elasticitetsenergin elektrostatiskt. Det betyder att, bortsett för lite friktionsförluster, deformationsprocessen inte ändrar strukturen på materialet. Arbetet som går åt för att deformera materialet lagras som potentiell energi eftersom avståndet mellan atomerna ändras från det normala. Temperaturen ändras inte i processen (fortfarande bortsett från friktion).

Gummi har en annan stuktur. Det består av polymerer i form av långa trådar. Trådarna är i viloläget lösa och svänger som gitarrsträngar. När man spänner ett gummiband kan trådarna inte svänga längre. Trådarna avger sin kinetiska energi i form av värme. Gummibandet blir varmt. När spänningen tas bort sker det omvända: värme omvandlas till ordnade svängningar hos trådarna. Gummit kyls alltså ner. Elasticitetsenergin i gummi lagras alltså termiskt såväl som elektrostatiskt.

Experiment:
Häng upp en liten vikt i ett gummiband. Värm gummibandet med en hårtork och observera vad som händer. Till skillnad från de flesta material så blir gummibandet kortare med ökande temperatur. En del av värmeenergin får polymertrådarna i gummit att svänga mer, vilket medför att gummit kontraherar.

Nedanstående video demonstrerar på ett indirekt sätt denna negativa expansionskoefficient för gummiband:

Se även Elasticity_(physics) .
/Peter E

Nyckelord: elasticitet [4]; *vardagsfysik [64]; utvidgning [8];

*

Materiens innersta-Atomer-Kärnor [15410]

Fråga:
Varför ökar volymen när vatten fryser till is?
/Veckans fråga

Ursprunglig fråga:
Varför ökar volymen när vatten fryser till is? Finns det något enkelt sätt att förklara detta för högstadieelever? Finns det andra ämnen som uppför sig som vatten?
/Mats I, Finnbacksskolan, Lycksele

Svar:
Vatten har många ovanliga egenskaper (se t.ex. den mycket trevliga (men omfattande) sajten Water Structure and Science ). Sajten A gentle introduction to water and its structure är lite mer lättillgänglig. De flesta av egenskaperna kommer sig av att vattenmolekylen är en dipol och därför bildar vätebryggor (se nedan).

Att fruset vatten har lägre densitet än flytande kan förklaras med att vattnet genomgår en strukturförändring när det fryser, se länk 1 och Water_(molecule)#Density_of_water_and_ice .

Intuitivt är det inte svårt att föreställa sig att vattenmolekylerna i flytande form kan packas mer effektivt än i fast form. I fast form är ju strukturen kristallin och molekylerna radas upp som H-O.-H-O.-H... (O.-H-O kallas för vätebrygga). Denna upplinjering av atomerna är lite slöseri men plats jämfört med om man som i vätskeformen kan packa molekylerna fritt, se nedanstående bild från länk 1 där man kan se att det finns outnyttjade luckor i den högra bilden som är is.

Den stora skillnaden i densitet mellan vatten och is (c:a 10%) är nog unik, men Wikipedia-länken ovan listar några fler ämnen vars densitet minskar när de stelnar (t.ex. vismut och kisel).

Egenskapen att is flyter är mycket viktig för livets utveckling och överlevnad. Om is hade haft högre densitet än vatten skulle ju sjöar och hav bottenfrysa i stället för att skyddas från att frysa av ett islager. Vattenlevande djur hade då haft svårigheter att överleva.

Vatten har som sagt många unika egenskaper som listas under länk 2. De mest påtagliga är hög smältpunkt, hög kokpunkt och mycket hög ångbildningsvärme. De flesta av dessa anomala egenskaper beror på att vattenmolekylen är en dipol, se fråga 15508 nedan. Se även fråga 17391 .



/Peter E

Nyckelord: vatten/is [49]; *vardagsfysik [64];

1 http://www.iapws.org/faq1/freeze.htm
2 http://www.btinternet.com/~martin.chaplin/anmlies.html

*

Kraft-Rörelse [14921]

Fråga:
Beror friktionskraften av kontaktytan?
/Veckans fråga

Ursprunglig fråga:
Fick en förfrågan om friktionskraften är areaberoende. Känns som att det borde vara så att större area ger större friktion. Fast om jag kollar på formeln friktionskraften = friktionskoefficienten*normalkraften finns inget som visar på att friktionskraften skulle vara areaberoende. Är friktionskraften areaberoende? Hur förklarar man det? Hur förklarar man att vilofriktionen är större än glidfriktionen?
/Marianne A, Ehrensvärdska gymnasiet, Karlskrona

Svar:
Marianne! Friktionskraften är ett mycket komplicerat fenomen när man djupdyker i det, men standardmodellen är mycket enkel: friktionskraften f är proportionell mot normalkraften N:

f = mN

där proportionalitetskonstanten m kallas friktionskoefficient. Som du ser är det inget beroende av arean. Att det är så kan man intuitivt förstå eftersom om vi t.ex. dubblar arean så blir normalkraften per ytenhet hälften så stor, så resultatet blir oförändrat. Proportionaliteten mellan friktionskraften och normalkraften kan man förstå om man tänker på att den reella kontaktytan (utgörs av några atomer som sticker ut) är ganska liten. Om man ökar normalkraften så kommer atomerna att flytta sig lite, och fler atomer kommer i kontakt med varandra. Detta gör att friktionen ökar. Att den ökar linjärt kan enklast betraktas som ett experimentellt faktum.

Bilderna nedan (från Hyperphysics, länk 1) visar vilofriktion (statisk) och glidfriktion (kinetisk). Den förra är som du säger större. Anledningen är komplex och beror av materialet, men det har att göra med att knöligheter fastnar i varandra när klossen ligger still, medan denna effekt minskar om klossen rör sig.

I den nedre figuren visas friktionskraften f som funktion av den drivande kraften F. Klossen väger 10 kg, så normalkraften är hela tiden c:a 100 N. Till vänster (i origo) är friktionskoefficienten noll (ingen drivande kraft att motverka). Friktionskoefficienten ökar när man går åt höger tills den blir 0.5. Då övervinns friktionskraften och klossen börjar röra sig. Observera att så snart klossen sätts i rörelse så minskar friktionskoefficienten till (i det här exemplet) 0.4.

Se vidare länk 1 och friction för mer om friktion än du någonsin vill veta .



/Peter E

Nyckelord: friktion [53]; *vardagsfysik [64]; friktionskoefficient [5];

1 http://hyperphysics.phy-astr.gsu.edu/hbase/frict.html

*

Värme [14561]

Fråga:
Varför ökar kroppens temperatur bara någon tiondels grad trots att det är 100 grader celsius i bastun?
/Veckans fråga

Ursprunglig fråga:
Varför ökar kroppens temperatur bara någon tiondels grad trots att det är 100 grader celsius i bastun?
/Saher T, Göteborg

Svar:
Egentligen fysiologi, men vi försöker svara ändå: kroppen har en mycket avancerad temperaturreglering (termostat), se det relativt avancerade dokumentet under länk 1. Det viktigaste för att hålla kroppstemperaturen nere i en bastu är svettning, som genom vattnets höga ångbildningsvärme (fråga 11076) mycket effektivt för bort energi från kroppen.
/Peter E

Se även fråga 11076

Nyckelord: *vardagsfysik [64]; *fysiologi [13];

1 http://home.student.uu.se/gude4911/T4/FFB/Temperaturreglering.pdf

*

Ljud-Ljus-Vågor [14470]

Fråga:
Hur fungerar en reflex?
/Veckans fråga

Ursprunglig fråga:
Hej jag skulle gärna ha svar på dessa frågor.

Hur fungerar en reflex? Är det skillnad på olika reflexer?
/Ted W, Fenestra s:t jörgen, Göteborg

Svar:
Hej Ted! En reflex fungerar så att den nästan oavsett varifrån ljuset kommer reflekterar det tillbaka i den riktning det kom. Det finns olika sätt att åstadkomma detta de vanligaste är en klotformig lins eller hörn gjorda av speglande material.

Linsen fungerar så att ljuset bryts till en punkt, reflekteras och går samma väg tillbaka. Det är så kattens lysande ögon uppkommer.

Hörnan (se snackset, länk 1) fungerar så att efter tre reflektioner går ljuset tillbaka i samma riktning det kom från. Bilden nedan visar principen något förenklat i två dimensioner.

Reflexer är alltså användbara om man använder en lampa som är nära ögat för att se när det är mörkt. En vanlig vit yta sprider det inkommande ljuset i alla riktningar, medan reflexen skickar allt ljus tillbaka till ljuskällan. Det är anledningen till att man ser reflexer så mycket bättre än även vita ytor.

Använd alltid reflex nör du är ute när det är mörkt! Tänk på att även om du ser en bil bra för att den har belysning, så om du inte har reflexer kanske bilföraren inte ser dig!

Se vidare Retroreflector .



/Peter E

Nyckelord: *vardagsfysik [64];

1 http://snacks.fysik.org/showSnack.asp?id=12

*

Värme [14395]

Fråga:
Varför kokar (stormkokar) tevattnet på nytt när jag doppar i tepåsen. Vattnet är först värmt i mikron tills det bubblar och sedan stilla innan påsen doppas.
/Lina D, Uggleskolan, Södra Sandby

Svar:
Hej Lina!

Vi har flera svar på detta i frågelådan, se t.ex. fråga 2458 nedan. Enkelt uttryckt så värms vattnet så effektivt i mikrovågsugnen att det inte hinner koka. Det blir i stället överhettat, dvs det blir varmare än den normala kokpunkten 100 grader.

Kokning innebär att det bildas bubblor av vattenånga som stiger till ytan. Det är emellertid inte helt lätt för vattnet att bilda bubblor - det kostar energi på grund av ytspänningen. Det är lättare om det finns en yta där småbubblor kan bildas. Det är därför det börjar bubbla så häftigt när du stoppar ner något i vattnet du just tagit ut ur mikrovågsugnen. Se vidare om kokande vatten och ytspänning .
/Peter E

Se även fråga 2458

Nyckelord: kokande vatten [17]; mikrovågsugn [25]; *vardagsfysik [64];

*

Värme [14174]

Fråga:
Hejsan! Jag undrar vad det är för vätska i kylklampar som gör att dom håller sig kalla länge... Och finns det nån annan vätska/vätskeblandning som har högre värmekapacitivitet som skulle hålla sig kallt ännu längre?
/Mikael H, Tullängsskolan, Örebro

Svar:
Såvitt jag vet innehåller kylklampar vatten eftersom vatten är ganska unikt för att det har mycket högt smältvärme (333 kJ/kg) och det är dessutom billigt och ofarligt. Jag känner inte till någon bättre vätska. Se även Ice_pack .

Addendum:

För att sänka temperaturen hos de nedkylda kylklamparna har man i stället för rent vatten en saltlösning. Eftersom saltlösningen sänker fryspunkten, så kommer de smältande klamparna att vara kallare än 0oC, vilket är fördelaktigt för kyleffekten. Man förlorar dock lite kylkapacitet eftersom smältvärmet blir mindre.

Tack Olof Pettersson för påpekandet!
/Peter E

Nyckelord: *vardagsfysik [64]; kokpunkts/fryspunkts förändring [14]; kylning [7];

*

Blandat [14013]

Fråga:
Jag tog ut en flaska vodka som legat i frysen, jag såg att det hade bildats en samling skivor av fryst vätska i vätskan, och jag undrar, består dessa flak, av vatten eller alkohol, eller en blandning av båda och varför?
/egon b, gbg

Svar:
Vatten blandat med alkohol fryser vanligtvis inte till en stor isklump utan till en is-sörja. Det beror på att alkohol+vatten har lägre fryspunkt än rent vatten. Vattenmolekyler kommer därför att frysas till is från slumpmässiga små isbitar. Dessa isbitar växer genom att ta upp mer vattenmolekyler och lämnar kvar en allt starkare blandning alkohol+vatten. Blandningen får därmed lägre fryspunkt, så det fordras ganska låga temperaturer för att frysa allt. Den påhittige läsaren kan säker räkna ut vad man kan använda den här effekten till !

Ämnet (Blandat) var ovanligt passande denna gång!
/Peter E

Nyckelord: kokpunkts/fryspunkts förändring [14]; *vardagsfysik [64];

*

Blandat [13516]

Fråga:
Hur kommer det sig att det analoga ljudet kommer fram snabbare än det digitala?
/Veckans fråga

Ursprunglig fråga:
Hur kommer det sig att det analoga ljudet kommer fram snabbare än det digitala? Är det så? När jag har två radioapparater på som sänder på de olika sätten så hör man den analoga sändningen först.
/Lisa S

Svar:
Det är samma sak med digital TV. Om du tittar på SVT1 digitalt i ett rum och analogt i ett annat, så ligger det digitala ljudet efter.

Nej, de elektriska signalerna för analogt ljud går inte snabbare än de för digitalt ljud. Fördröjningen beror på att elektroniken som förvandlar de digitala signalerna till ljud du kan höra (analogt ljud, apparaten kallas en digital-till-analogomvandlare) tar lite tid för att utföra den uppgiften. Det analoga ljudet kan däremot (via en förstärkare som ger liten fördröjning) direkt driva högtalaren. Så även om signalerna gått samma sträcka hör du det analoga ljudet tidigare.

Sedan kan vägen signalerna går variera: det digitala kanske tar en omväg via en geostationär satellit . En sådan tar c:a 0.3 sekunder. Ytterligare en effekt som kan fördröja signalen är om de går igenom t.ex. en DVD-spelare. Det kan ta lite tid för elektroniken i denna att sända ut signalen igen.

Du kan ibland märka en annan irriterande effekt: att kommentatorn ligger lite före bilden och kommenterar det du ännu inte sett (t.ex. att bollen gick i mål). Detta beror på att bilden gått via en eller flera geostationära satelliter som befinner sig på ett avstånd av c:a en tiondel av månens avstånd. Kommentatorljudet går ofta via telefonledningar som går längs jordytan. Ljudet har alltså betydligt kortare väg och kommer alltså fram före bilden. Detta borde "synkas" innan den sammansatta signalen sänds ut, men detta slarvar man ofta med.
/Peter E

Nyckelord: *vardagsfysik [64];

*

Kraft-Rörelse [12887]

Fråga:
Hur kan ett stort fartyg som är gjort av plåt och järn och är fullt med last flyta?
/Veckans fråga

Ursprunglig fråga:
Hur kan ett stort fartyg som är gjort av plåt och järn och är fullt med last flyta?
/Lisa T, Marmaverkans, Söderhamn

Svar:
Lisa! Det är Arkimedes förtjänst! Nåja, han förklarade det åtminstone. Nationalencyklopedin säger om Arkimedes princip: en lag i hydrostatiken som säger att en kropp som är helt eller delvis nedsänkt i en vätska påverkas av vätskan med en uppåtriktad kraft vilken till sitt belopp är lika med den undanträngda vätskans tyngd.

Det är medeldensiteten som är avgörande, inte densiteten av det material farkosten är gjord av. Föreställ dig en innerslang av gummi. Om den inte innehåller någon luft, så sjunker den i vattnet. Om du pumpar luft i slangen så väger den lika mycket (den väger i själva verket lite mer), men volymen ökar mycket. Densiteten är ju massa/volym, så densiteten minskar. När densiteten blir mindre än vattnets densitet så flyter slangen.

Det är samma sak med en båt. Även om båten är gjord av järn (som har hög densitet och sjunker), så finns det utrymmen i en båt som innehåller luft (t.ex. kaptenens hytt). Så länge vattnet hindras komma in i dessa utrymmena flyter båten. Om det däremot kommer in vatten, t.ex. genom att båten kantrar, så sjunker båten.

Se även frågorna 8194 och 15378 .



/Peter E

Nyckelord: Arkimedes princip [32]; *vardagsfysik [64];

1 http://science.howstuffworks.com/question254.htm

*

Ljud-Ljus-Vågor [12744]

Fråga:
Om man blåser i ett glasrör med vatten sätts ju luften i svängning och längden på luftstapeln bestämmer tonhöjden - ju mindre vatten desto lägre ton. Om man slår på glasröret blir det tvärtom - ju mindre vatten desto högre ton. Jag antar att det är glaset självt som kommer i svängning och att vattenmängden ändrar dess frekvens. Hur går det egentligen till? Finns det någon formel med vilken man kan räkna ut frekvensen beroende på glasets egenskaper?
/Josefin R, Kubikskolan, Malmö

Svar:
Josefin! Det är riktigt att det är glaset som kommer i svängning när du slår på det. Vattenhöjden i bottnen reglerar längden på den del av glasröret som kan svänga fritt, och detta påverkar precis som du observerat frekvensen på ljudet man hör. (Glasröret sätter ju den omgivande luften i rörelse så att vi kan uppfatta ljudet.) Detta är precis samma sak som när man spänner in en metallstav och knäpper på den - den fria längden och metallens egenskaper bestämmer frekvensen: kort stav - hög frekvens och lång stav - låg frekvens.

Om du i stället blåser i röret så fungerar glasröret som en orgelpipa: du får en stående våg med en nod (ingen vibration) vid vattenytan och en buk (maximal vibration) i ändan av röret, se fråga 10178 . Om rörets längd från vattenytan till ändan är L ges våglängden l av

l = 4*L

Frekvensen blir då

f = vljud/l = vljud/(4L)

dvs frekvensen avtar med ökande L. Om L = 0.5 m får vi

f = (340 m/s)/(4*0.5 m) = 170 Hz (s-1)

Du efterlyser ett samband med vilken man kan få ut ljudfrekvensen som en funktion av glasets egenskaper, och det är nu det blir svårare! Trots ett ivrigt letande i diverse böcker och på nätet har jag nämligen inte kunnat hitta någon fix och färdig formel - som så ofta i fysiken står vi inför en situation som är mycket svår att beskriva teoretiskt... Men det ger ju istället upphov till en intressant experimentell utmaning, nämligen att själv försöka hitta det önskade sambandet med hjälp av egna mätningar och observationer!

Om man funderar ett tag på vad som kan spela in, kommer man fram till att rörets diameter, tjocklek och längd kan vara viktiga. Glas är ett ganska spännande material eftersom det har en speciell "oordnad" struktur - man säger att det är "amorft" - och det kan därför tänkas att glasets kemiska sammansättning liksom dess temperatur också har stor betydelse för vilken egenfrekvens som glasröret får.

Se emellertid fråga 13398 . Där får man till synes det motsatta resultatet när man drar fingret längs glaskanten. Anledningen till skillnaden är att i det halvfulla glaset svänger både glaset och vattnet. I glasröret ovan fungerar vattnet som en dämpning och bara glaset vibrerar.

Pröva själv: hitta glasrör med olika diameter & tjocklek och, om det är möjligt, gjorda av olika sorts glas. Koppla upp en mikrofon till ett oscilloskop (eller en frekvensanalysator om du har tillgång till en sådan) så att du kan mäta frekvensen med relativt god noggrannhet. Variera vattenpelarens höjd stegvis och gör upp en tabell. Genom att arbeta systematiskt och bara variera en glasrörsparameter i taget kan man nog åtminstone komma fram till vilka egenskaper hos glasröret som har störst betydelse.

Om du eller någon annan "därute" genomför experimentet blir vi jätteglada om ni hör av er till Frågelådan och berättar vad resultatet blev!



/Margareta H/lpe

Nyckelord: ljud, resonans [19]; *vardagsfysik [64]; orgelpipa [4];

*

Ljud-Ljus-Vågor [12640]

Fråga:
Varför vänds spegelbilden höger/vänster och inte upp/ned?
/Veckans fråga

Ursprunglig fråga:
Jag har funderat på en sak med speglar. Varför vänds spegelbilden höger/vänster och inte upp/ned?
/Emma B, Göteborg

Svar:
Emma! Detta är en bra och klassisk fråga. Det finns många felaktiga svar, varav att anledningen är att ögonen sitter till höger och vänster är ett. Stäng ett öga! Du ser samma bild! Det har alltså inget med ögonens placering att göra!

Detta är faktiskt en kuggfråga. En spegel kastar inte om riktningen i sidled, lika lite som den kastar om upp och ned, det är vi som gör det. Däremot kastar den om riktningen vinkelrätt mot spegelytan.

Tänk dig att vi sätter upp en stor spegel längst fram på tavlan i ditt klassrum. En lärare kommer in, ställer sig framför spegeln och pekar uppåt. Spegelbilden pekar uppåt. Läraren pekar neråt - spegelbilden pekar neråt. Läraren pekar mot fönstret - spegelbilden pekar mot fönstret. Läraren pekar mot dörren - likaså spegelbilden. Detta är absoluta riktningar, och ändras inte av en spegel. Vänster och höger är däremot relativriktningar som mäts jämfört med en person. En verklig person som står mittemot dig har roterat i förhållande till dig och har därför annan uppfattning om vad som är höger och vänster.

Experiment: Ta ett halvgenomskinligt papper och skriv ditt namn på det. Håll upp det med framsidan mot spegeln. Vad ser du? Ditt namn är i spegeln skrivet från höger till vänster med bakvända bokstäver. Men det ser precis ut som det du kan läsa genom pappret. Vänd nu pappret så att framsidan är mot dig. Vad ser du nu?

Nedan är två länkar med korrekta förklaringar på engelska.
/Peter E

Nyckelord: spegel [10]; *vardagsfysik [64];

1 http://science.howstuffworks.com/question415.htm
2 http://www.efn.org/~danrob/tech/leftrite.htm

*

Kraft-Rörelse [12527]

Fråga:
Snurr på kontorsstol
/Veckans fråga

Ursprunglig fråga:
Man snurrar på en kontorsstol, sträcker man sedan ut benen går det saktare pga av luftmotståndet. Men om jag sedan drar in benen går det återigen snabbare. Hur kan det komma sig?
/Martin

Svar:
Intressant fråga eftersom din förklaring att det är luftmotståndet som bromsar är fel. Om det varit luftmotståndet så har du helt rätt: hastigheten hade inte ökat när du drar in benen. Du ser samma effekt när en konståkare gör en piruett, se bilden.

Förklaringen är att något som kallas rörelsemängdsmoment L - produkten av en kropps tröghetsmoment I och dess rotationshastighet w (vinkelhastighet) L=Iw - måste bevaras (Se Rörelsemängdsmoment ) och (Angular_momentum ).

Se snackset Rotera mera och fråga 9154 .



/Peter E

Nyckelord: rörelsemängdsmoment [14]; *vardagsfysik [64];

*

Blandat [12321]

Fråga:
Varför är katter gråa i mörkret?
/Veckans fråga

Ursprunglig fråga:
Varför är katter gråa i mörkret? Varför ska man fokusera sin blick vid sidan av det föremålet man vill se på i mörker?
/stina a, fristadshögstadie, fristad

Svar:
Det beror på hur ögat är konstruerat. Synceller består av två typer: tappar som finns i tre varianter och kan se färger och stavar som inte ser färg men som är känsligare.

På natten fungerar bara stavarna eftersom tapparna kräver mer ljus. Du är därför färgblind på natten.

Svaret på din andra fråga är att det centralt på näthinnan finns mycket tappar och färre stavar. Detta för att ge bra färgseende. I utkanten av näthinnan finns många stavar, dvs mörkerseendet är bättre där.

Observera att stavarna är inaktiverade när belysningen är god. Det tar flera minuter i mörkret innan du ser bra. Detta kallas ögats mörkeradaption. Det finns andra intressanta egenskaper vad gäller synfunktionen hos människan, se förklaringen till Machs band i Mach bands .

Man kan fråga sig varför ögat av evolutionen utvecklats på detta sätt med stavar som är färgblinda. Det beror ganska säkert på att stavarna kan göras känsligare (är mer effektiva detektorer) eftersom de kan detektera allt infallande ljus medan tapparna bara detekterar ljus av en viss våglängd.
/Peter E

Se även fråga 3139

Nyckelord: färg/färgseende [39]; *vardagsfysik [64]; #ljus [63];

*

Ljud-Ljus-Vågor [12225]

Fråga:
Hur tar polaroidglasögon bort reflexer?
/Veckans fråga

Ursprunglig fråga:
När ljus reflekterat svänger det bara i ett plan och därför kan vi slippa reflexer med polaroidglasögon. Jag förstår hur glasögonen fungerar, men inte varför reflexionen gör ljuset polariserat. Hoppas ni kan hjälpa mig med det.
/Eva B, Sandagymnasiet, Huskvarna

Svar:
Vid reflexion ändras både ljusets intensitet och polarisation. När en ljusstråle träffar gränsytan på ett genomskinligt medium kommer en del av ljuset att brytas och en del av ljuset att reflekteras. Om ljuset faller in mot gränsytan så att den brutna strålen och den reflekterade strålen är vinkelräta mot varandra, så kommer det reflekterade ljuset att vara planpolariserat. Infallsvinkeln som uppfyller sambandet, och alltså ger 100% polariserat reflekterat ljus, kallas för brewstervinkeln.

Om det i brewstervinkeln infallande ljuset är planpolariserat vinkelrätt mot infallsplanet kommer allt ljus att brytas ner i mediet med högre brytningsindex och inget ljus reflekteras.

Vi tar en mycket enkel modell av spridningen, se nedanstående figur: den inkommande vågens E-fält (som definieras som polarisationsriktningen) sätter elektroner i svängning i gränsytan. Komponenten som är parallell med normalen får elektronerna att svänga nästan i den riktning som den reflekterade strålen går (exakt för Brewstervikeln). En dipol strålar inte i denna riktning. Det betyder att den med infallsplanet parallella komponenten inte reflekteras, utan måste gå in i mediet. Den andra komponenten - som reflekteras utmärkt - tas om hand av polaroidglasögonen. Alltså ser vi inget reflekterat ljus.

Se vidare Brewster's_angle .



/Peter E

Nyckelord: polaroidglasögon [3]; *vardagsfysik [64]; polarisation [7]; #ljus [63];

*

Ljud-Ljus-Vågor [11883]

Fråga:
Ställer mig tveksam till svaret på fråga 11865. Att knäppet i bakplåten hörs är väl snarare en effekt av att ljuset värmer upp luften som utvidgas och "knäpper" bakplåten. Blixten måste nämligen hållas nära bakplåten. En blixt som hålls längre ifrån (och där försås inte ljuset sprids åt sidorna) skulle inte ge något knäpp.
/Marcus P, Örebro

Svar:
Din kommentar till fråga 11865 har väckt tvivel i mitt sinne, så jag gjorde ett experiment för att undersöka saken. Jag sotade med en stearinljuslåga ena sidan av en bit aluminiumfolie av en handflatas storlek. När man sotar aluminiumfolie på detta sätt, måste man hela tiden hålla den i snabb rörelse, annars smälter den. Vad kan vi nu vänta oss om vi bränner av en fotoblixt mot folien?

1. Knäppet uppstår genom impulsöverföring (ljustryck). Ljus som reflekteras överför dubbelt så mycket impuls som ljus som absorberas. Det bör knäppa starkare när man blixtrar mot den speglande ytan.

2. Knäppet uppstår genom snabb uppvärmning af luften. En aluminiumyta reflekterar ungefär 95 % av ljuset, medan sotet absorberar nästan allt ljuset. Det bör knäppa starkast när man blixtrar mot sotet.

Det hördes ett starkt knäpp när jag blixtrade mot sotet, men inget alls när jag blixtrade mot den blanka ytan. Det är alltså 2. som gäller, alltså att din invändning är helt befogad. Sotet värms upp av den snabba blixten. En del av värmen överförs till luften som expanderar. Det knäpper.

Vi tackar för din kommentar, och ska nu rätta till de ställen där vi givit vilseledande svar.

Tillägg 30/11/2016: (LPE)

Här är en video med ett liknande försök:

Se Crookes_radiometer , bilden nedan, för detaljerad förklaring av ett liknande försök.



/KS/LPE

Se även fråga 11865 och fråga 20427

Nyckelord: knäppande aluminiumfolie [2]; #ljus [63]; *vardagsfysik [64];

*

Värme [11227]

Fråga:
hej! jag håller på att skriva projektarbete om vardagens naturvetenskap. man kan ju kyla en flaska med vätska genom att vira en handduk runt och lägga den i solen. men hur mkt går att kyla? ex om det är 25 grader varmt ute och man vill ha flaskan kylskåpstemp (ca +6) går det? vad ska vattnet vara för temp? måste man byta vatten i handduken? hur mycket energi tar den från luften gentemot handduken? material i flaska/burk spelar roll och formen. aluminiumplunta ultimat? tack på förhand!! mvh/ Johanna
/Johanna w, Dragonskolan, Umeå

Svar:
Det är avdunstningen som kyler, och den beror på luftfuktigheten, temperaturen och vindhastigheten. Är luftfuktigheten 100 % (dimma) blir det ingen avkylning alls. Det finns inga enkla formler för detta problem. Det bästa är att pröva själv. Luftfuktigheten kan mäta med en assmanhygrometer, som består av en våt och en torr termometer, kolla sajten.
/KS

Nyckelord: kylning [7]; *vardagsfysik [64];

1 http://www.taftan.com/thermodynamics/HYGROMT.HTM

*

Ljud-Ljus-Vågor [2391]

Fråga:
Varför skimrar plastfodralen till kassettband i olika färger?
/Veckans fråga

Ursprunglig fråga:
Varför skimrar plastfodralen till kassettband i reflekterat ljus?
/erik w, östersund

Svar:
Du ser denna effekt så snart du har att göra med tunna genomskinliga skikt, som t.ex. plastfodral, såpbubblor eller ett tunnt lager bensin på vattnet.

Det beror på att en del av ljuset går igenom det tunna skiktet och reflekteras, medan en del reflekteras redan i den första ytan. Detta två ljusstrålar interfererar med varandra så att vissa färger förstärks och vissa släcks ut beroende på om vägskillnaden är ett jämnt antal våglängder eller ej. Eftersom skiktets tjocklek ofta varierar i olika punkter, så ser du ett färgat mönster.

Vad händer då med fotonerna (ljuspartiklarna) som släcks ut? Försvinner de bara? I så fall skulle lagen om energins bevarande inte vara uppfylld!

Ingen fara! Ljuset (energin) kan aldrig försvinna genom interferens. Vad som sker är att man får en omfördelning av strålningen, dvs de fotoner som saknas i de mörka områdena har i stället gått ut i en annan riktning. Antalet fotoner (energin) är alltså konstant, medan fördelningen i olika riktningar ändras genom interferensen.

Vissa fjärilar åstadkommer ett nästan självlysande intryck med hjälp av fjäll av exakt anpassad tjocklek, se länk 1:



/Peter Ekström

Nyckelord: interferens [14]; *vardagsfysik [64]; #ljus [63];

1 http://animals.howstuffworks.com/insects/butterfly-colors.htm

*

Ljud-Ljus-Vågor [10178]

Fråga:
Vad är det som händer med rösten när man andas helium?
/Veckans fråga

Ursprunglig fråga:
Vad är det som händer med rösten när man andas helium?
/jan w, sommarhämsskolan, uddevalla

Svar:
Låt oss se hur en orgelpipa med längden L fungerar. Eftersom pipan är stängd i ena ändan och öppen i den andra, så är den fundamentala (längsta) våglängden l=4*L, eftersom vi måste ha en nod (ingen svängning) i den slutna ändan och en antinod (maximal svängning) i den öppna.

Det är viktigt att komma ihåg, att våglängden bestäms av pipans längd, och alltså är oberoende av vilken gas man blåser med. Ljudhastigheten är däremot olika i olika gaser. Ljudhastigheten i helium är 2.6 gånger högre än luftens.

Ljudhastigheten är proportionell mot molekylernas medelhastighet, som i sin tur är proportionell mot roten av absoluta temperaturen. En typisk luftmolekyl (kväve) är 7 gånger tyngre än en heliummolekyl (28/4 = 7). Vid en given temperatur är medelenergin för luftmolekylerna lika med medelenergin hos heliummolekylerna. Ur det får vi att kvoten mellan ljudhastigheterna blir:

(7)1/2 = 2.6

För alla vågrörelser gäller sambandet att utbredningshastigheten är våglängden*frekvensen, dvs v=l*f. Frekvensen som örat uppfattar blir alltså f=v/l= v/4L. Eftersom ljudhastigheten i helium är 2.6 gånger så hög som ljudhastigheten i luft, så kommer en heliumfylld orgelpipa att ha 2.6 gånger högre naturlig frekvens än samma pipa med luft. Människans talorgan fungerar ungefär som en orgelpipa, och det är därför rösten ändras till högre frekvenser när man andas in helium och pratar.

I fråga 15873 finns en demonstration av vad som händer om man i stället andas in en tung gas som svavelhexafluorid.
/PE/KS

Nyckelord: ljud, resonans [19]; *vardagsfysik [64]; orgelpipa [4];

*

Värme [10222]

Fråga:
Om man blåser på huden när det är varmt ute så känns luften sval och skön. Men om man blåser på huden när man sitter i en varm bastu så känns luften het. Varför är det så?
/Anton F, Geresta, Härnösand

Svar:
I båda fallen finns ett tunt (kanske 1 mm) luftskikt utanför huden som har en temperatur på gissningsvis 30 oC. Luft leder värme dåligt, så detta skikt verkar isolerande. Om det utomhus är 25 oC och du blåser på huden, blir det ju kallare. I bastun blir det varmare, därför att du ersätter det tunna skiktet med varmare luft.

Det är lite mera komplicerat än så här. Luftfuktighet och avdunstning påverkar också.
/KS

Se även fråga 2828

Nyckelord: *vardagsfysik [64];

*

Kraft-Rörelse [8366]

Fråga:
En interstellär farkost kan förses med ett solsegel. Hur ska ett sådant segel fungera ? Har det någon betydelse om ytan som är vänd mot solen är blank eller matt ? Motivera ditt svar. En liten snurra med små metallplåtar är upphängd i ett vakuumrör. Metallplåtarna är blanka på den ena sidan och matt svarta på den andra. När man lyser på den snurrar den som om den matta ytan fick mest rörelsemängd från fotonerna. Hur går detta ihop? Hasse!
/Hans D, Cederbladsskolan, Uppsala

Svar:
En foton som reflekteras avger dubbelt så mycket rörelsemängd som en foton som absorberas. Den välbekanta apparat du beskriver (se nedanstående bild), tycks då snurra åt fel håll.

Nu är det faktiskt inte vakuum där inne, utan gas vid lågt tryck. De svarta ytorna blir varmare än de speglande eftersom de absorberar mer ljus. När en gasmolekyl kolliderar med en svart yta, får den en extra spark. Det blir alltså i medeltal större rekyl på de svarta ytorna. Det är det som driver rotationen. Hade vi lyckats åstadkomma tillräckligt bra vakuum, skulle den (i princip, men effekten är för liten) snurrat åt andra hållet.

Apparaten beskrivs i detalj i Wikipedia-artikeln Crookes_radiometer .



/Peter E/KS

Nyckelord: *vardagsfysik [64];

*

Värme [7830]

Fråga:
Frysning av vatten
/Veckans fråga

Ursprunglig fråga:
Om man tar 50 gradigt vatten i en spann och ställer ut i -10 gradig luft samtidigt med en spann med 20 gradigt vatten. Vilken fryser till is först?. Har hört att "varmt" vatten fryser fortast men har svårt att tro på det.
/Claes L, Härryda

Svar:
Det är klart att den kallare i de flesta fall fryser först. Effekten är emellertid relativt väletablerad men utan någon enskild enkel förklaring, se Mpemba_effect .

Nedanstående länkar diskuterar några möjliga förklaringar till effekten.

Eftersom det avsvalnande varma vattnet vid någon tidpunkt måste vara av samma temperatur som det kalla vattnet så är det svårt att se att det från början varma vattnet skulle frysa snabbare: två vattenmassor med samma temperatur bör rimligen uppföra sig likadant. Det måste i så fall vara någon skillnad på vattenmassorna, t.ex. gasinnehåll.

En av förklaringarna går ut på att det varma vattnet fryser delvis snabbare (mindre underkylning, se fråga 16785 ) och därför uppfattas som fruset när det i själva verket bara är delvis kylt. Problemet kvarstår dock: vad är det för skillnad mellan det kalla vattnet från början och det varma vattnet när temperaturen passerar begynnelsetemperaturen av det kalla vattnet.

Men det hjälper inte att spekulera ... man borde göra ett försök att påvisa effekten! Jag gjorde ett enkelt försök med istärningsbehållarna. Det var ingen märkbar skillnad på behållaren med kallt vatten och behållaren med hett vatten. Om något så frös den varma vattnet långsammare!

Se även frågorna 18157 , 3551 och 16785 .

Påpekas bör dock att effekten inte är absolut bevisad, se Mpemba_effect#Recent_views .

Ett antal möjliga förklaringar listas i Mpemba_effect#Suggested_explanations . Jag tror, om effekten finns, att förklaringen är att det från början kalla vattnet har en större tendens till underkylning.

Försök med temperaturmätning

I länk 2 beskrivs ett experiment där man mätt temperaturen som funktion av tiden, se nedanstående figur. Experimentet är emellertid mycket bristfälligt dokumenterat (t.ex. Var placerades temperatursensorerna? Vad var omgivningens temeratur?).

I början ser plotten rimlig ut. Den röda kurvan (varmare vatten) är brantare än den blå. Detta är rimligt eftersom värmetransporten till omgivningen är proportionell mot temperaturskillnaden.

Från 10 grader är kurvorna i stort sett parallella, vilket är rimligt. Kurvorna planar emellertid ut vid +3 grader och inte som man väntar sig vid vattnets fryspunkt 0 grader. Detta är antingen ett kalibreringsfel eller så sitter temperatursensorerna på fel plats.

Om man verkligen kan tro på mätningen så är det uppenbart att förklaringen är att tiden för frysning är kortare för det från början varma vattnet. På något sätt måste värmetransporten för det varma vattnet vara effektivare än för det kalla vattnet. Vi har fullständig isbildning (kurvan böjer nedåt och isen kyls till under 0 grader) vid 2.5 timmar för det varma vattnet och 4.2 timmar för det kalla vattnet.

Detta är ju en mycket tydlig effekt om man kan lita på mätningen. Varför 0-gradigt vatten som varit varmt fryser snabbare ges emellertid ingen förklaring för.

Se emellertid länkarna

https://www.sciencenews.org/article/debate-heats-over-claims-hot-water-sometimes-freezes-faster-cold

https://medium.com/the-physics-arxiv-blog/why-hot-water-freezes-faster-than-cold-physicists-solve-the-mpemba-effect-d8a2f611e853

där förklaringen föreslås vara att fler vätebindningar bildas vid den högre temperaturen. Hur detta påverkar kylhastigheten är emellertid inte uppenbart.



/KS/lpe

Nyckelord: vatten/is [49]; *vardagsfysik [64]; värmeöverföring/transport [46]; Mpemba-effekten [2];

1 http://www.rsc.org/images/nikola-bregovic-entry_tcm18-225169.pdf
2 https://www.picotech.com/library/results/freezing-hot-cold-water

*

Värme [7130]

Fråga:
Hej! Varför behöver man inte skrapa rutorna på bilen när den står inne i carporten på natten? Det är ju nästan lika kallt (=också minusgrader) där som det är utomhus?
/Martin T, Fagrabäcksskolan, Växjö

Svar:
Vi förutsätter att det är klart väder. Taket är varmare än natthimlen. Det kommer alltså mer värmestrålning från taket än från himlen. Om du mäter glasrutans temperatur i de två fallen, finner du att den är lägre under bar himmel eftersom nettoutstrålningen (strålning ut - strålning in) blir större.

Alla vet att det är det kallare när det är klart än när det är molnigt? Samma anledning - mindre strålning in eftersom den högre atmosfären är kallare! Om vi dessutom tar hänsyn till att en molnfri atmosfär har lite materia som strålar, kan man säga att natthimlens temperatur är 2.7 K - temperaturen hos den kosmiska bakgrundsstrålningen!

Samma effekt gör att snön smälter snabbare under ett träd än utanför - området nära trädet är varmare.
/KS/lpe

Se även fråga 13747 och fråga 14371

Nyckelord: strålning, in-/ut- [6]; *vardagsfysik [64]; *meteorologi [20];

*

Elektricitet-Magnetism [5019]

Fråga:
Jag har en f.d. kock i min fysik B på basåret. Han har använt en induktionsspis. Den ser enligt honom ut som en "vanlig" spis med keramikhäll. Kastrullen kan möjligen vara lite mer tjockbottnad än en vanlig kastrull. När man tar av kastrullen efter kokning är "plattan" ej varm. Hur fungerar en sådan induktionsspis?
/Jan-Erik E, Komvux, Östersund

Svar:
Under ytan finns en stor spole genom vilken man skickar en högfrekvent växelström. Då alstras ett högfrekvent magnetfält. Placerar man en metall i detta fält induceras virvelströmmar i denna metall, och genom ohmska förluster värms metallen upp. Nu har expertis meddelat oss att det måste finnas en järnplatta i kastrullen. Det innebär att det inte bara är induktion som är inblandad, utan också det man i transformatorsammanhang kallar järnförluster. Det kan också vara så att järnet hjälper till att koncentrera magnetfältet till kastrullen. Enligt vår sagesman förekommer induktionsspisar knappast i hemmen, det är proffsutrustning.

Se vidare länk 1 och 2, Induktionshäll och Induction_cooker .
/KS/lpe

Nyckelord: *vardagsfysik [64]; induktion [13];

1 http://home.howstuffworks.com/induction-cooktops.htm
2 http://www.explainthatstuff.com/induction-cooktops.html

*

Blandat [62]

Fråga:
När man tänder en lampa med lysrör blinkar den ett par sekunder innan den får sitt konstanta sken. (Att jämföra med en vanlig glödlampa som tänds direkt.) Varför?
/

Svar:
Glödlampor och lysrör fungerar på helt olika sätt. I glödlampan är det en tråd som upphettas så att den glöder av en ström. Detta sker mycket snabbt.

I lysröret så sker en urladdning av elektroner som rusar genom en tunn gas. När elektronerna stöter på gasatomerna så blir dessa exciterade och utsänder ljus. Detta ljus omvandlas sedan till andra våglängder i ett (fluoriscerande) skikt på lysrörets insida. Det tar en viss tid innan urladdningen i lysröret har stabiliserats och då kan det bli blinkningar. En del typer av lågenergilampor fungerar i princip som lysrör.

Undersök: Med hjälp av ett gitter eller en enkel spektrometer kan man undersöka ljuset från olika lampor. Vad är det för skillnad på ljuset från en vanlig lampa och från ett lysrör?

Nyckelord: lysrör [10]; lågenergilampa [13]; *vardagsfysik [64];

*

Blandat [64]

Fråga:
Varför åker duschdraperiet mot en, när man duschar??
/

Svar:
Mycket trevlig fråga. Vi är nog många som gjort denna iakttagelse. Denna fråga har lett till omfattande diskussioner och försök beroende på att det finns två olika förklaringar:

Den första går ut på att duschen drar med sig luften så att det blir en nedåtgående luftström längs draperiets insida. Man får då samma effekt som på en flygplansvinge: På ovansidan där luften strömmar snabbare där är trycket lägre (bernoullieffekten).

Pröva: Tag en tesked och håll lätt i den längst ut i skaftet. För in baksidan av skeden i en vattenstråle från en vanlig kran och känn hur skeden "dras in" i strålen.

Den andra förklaringen kan vi kalla för "skorstenseffekten". När Du duschar så uppvärms luften inne i duschen och denna varma luft stiger varvid trycket sjunker längst ned.

Pröva: Du kan testa teorierna själv genom att duscha med olika varmt (kallt) vatten!

Se vidare Bernoulli's_principle .

Bilden nedan (från Wikimedia Commons ) visar tydligt att trycket i den vänstra delen av röret är lägre eftersom luften där rör sig snabbare (röret är smalare). Apparaten kallas Venturirör och används för att mäta gasflöde. En liknande anordning används för att mäta flygplans hastighet, se Pitot_tube .



/GO/lpe

Nyckelord: duschdraperi [2]; bernoullieffekten [6]; *vardagsfysik [64];

*

Ljud-Ljus-Vågor [793]

Fråga:
Vart tar ljuset vägen när man släcker lampan ?
/Veckans fråga

Ursprunglig fråga:
Om man har ett ljusisolerat rum med en lampa i, var tar ljuset vägen när man släcker lampan ?
/Sandra A, Ölyckeskolan, Löberöd

Svar:
Låt oss först titta på "ett vanligt rum". Ljuset strålar hela tiden ut från lampan och träffar väggarna där det mesta "sugs upp" av väggen och resten reflekteras. Om man släcker lampan så dröjer det inte länge förrän allt ljuset "sugits upp" av väggarna. Energin i ljuset övergår i värmeenergi i väggarna.

Du säger ljusisolerat rum och med det menar Du säkert ett rum som har speglar på alla väggarna och även på golvet och i taket. Om man släcker ljuset i ett sådant rum så skulle ljuset studsa fram och tillbaka för all framtid om speglarna vore helt perfekta. I verkligheten är de inte det – vid varje reflektion "sugs" lite av ljuset upp och det dör ut efter hand.

Observera att det ljus du ser (det som hamnar i ögat) försvinner i ögat. Det betyder att även om du har perfekta speglar så kan du inte observera ljuset utan att förstöra det,

Ljus som färdas ut i världsrymden kommer att fortsätta i oändig tid om det inte träffar på något. Kommer rymden då att bli "ljusare"? Nej, pga universums expansion gör den inte det. Se vidare Olbers paradox .
/GO/lpe

Nyckelord: *vardagsfysik [64]; #ljus [63];

*

Värme [1166]

Fråga:
Avsvalnande kaffe
/Veckans fråga

Ursprunglig fråga:
Jag har fått en fråga, som jag själv inte kan besvara. Kan ni? "Jag ska just börja dricka mitt kaffe när telefonen ringer. Kaffet är redan upphällt och jag funderar om jag ska hälla i mjölken nu eller efter samtalet, om jag vill ha kaffet så varmt som möjligt. Hur ska jag göra?"
/Cecilia H, Mitthögskolan, Östersund

Svar:
Kaffet svalnar genom värmeledning till omgivningen. Det innebär att "avsvalningstakten" är större ju varmare kaffet är - värmetransporten är proportionell mot temperaturskillnaden mellan kaffet och omgivningen.

Om du först häller i mjölken så leds inte så mycket värme bort som om kaffet "varit för sig själv". Häll alltså i mjölken så fort som möjligt. Viktigare är att hindra värmet från att lämna koppen. Detta kan man göra dels genom att sätta koppen på ett isolerande underlag och dels genom att sätta lock på koppen. Fatet gör nog mer nytta ovanpå än under koppen!

Experiment: Varför inte undersöka detta experimentellt med två kaffekoppar!

Tillägg 9/1/08:

Detta är en klassisk fråga som diskuterats mycket. Frågan under länk 1 får samma svar men frågaren påstår sig ha mätt temoperaturer och kommit fram till motsatt resultat. Om detta kan man säga två saker:

1 Det är inte helt lätt att skapa identiska förhållanden och att mäta temperaturerna tillräckligt exakt.

2 Resonemanget bygger helt på att värmeförlusten sker genom ledning så att kyleffekten är proportionell mot temperaturskillnaden kaffe-omgivning. Vid rimliga temperaturer bör förlusterna genom strålning vara försumbara. (En ideal termos skulle t.ex. bara ha strålningsförluster och vi vet att kaffet håller sig mycket länge varmt i en termos. Se vidare fråga 14327 .)

Diagrammet nedan från illustrerar resonemanget mycket väl. Observera dels att "temperaturhoppet" när man häller i mjölken är samma för de båda fallen. Resultatet bygger då på att den blå avsvalningskurvan är mindre brant än den röda eftersom kaffet är kallare.



/GO/lpe

Nyckelord: värmeöverföring/transport [46]; *vardagsfysik [64];

1 http://www.madsci.org/posts/archives/2000-09/969405728.Ph.r.html

*

Ljud-Ljus-Vågor [841]

Fråga:
Varför är himlen ibland blå och ibland röd?
/Veckans fråga

Ursprunglig fråga:
Hej. Om det är av samma orsak som himlen ibland är röd som den ibland är blå, borde den då inte också lika ofta vara grön, då grön ligger mellan blå och röd i färgspektrat. Vad beror annars de olika färgerna på?
/Mats L, Pauliskolan, Malmö

Svar:
Mycket intressant fråga! Svaret är att himlen kan vara grön, det är bara det att det är sällsynt. Låt oss försöka reda ut de olika färgerna:

Mitt på dagen är himlen blå. Det beror på att solljuset sprids av luften, och blått ljus sprids mer än rött och gult. Eftersom det är spritt ljus, så är det också polariserat.

Försök: Sätt på dig ett par polarisationssolglasögon. Titta på en punkt 90 grader (ett kvarts varv) från solen. Vrid på huvudet. Kan du se att himlen blir mörkare när huvudet är i ett visst läge?

När solen håller på att gå ner, så har solljuset så lång väg att gå igenom atmosfären, att det blå ljuset är helt försvunnet (spritts bort). Det enda du ser då är det röda ljuset som sprids - du har en vackert röd solnedgång.

I ett mellanläge kan man ibland se att himlen är grön, men det är, som sagt, ganska ovanligt.

Se även snackset Varför är himlen blå? .
/Peter Ekström

Nyckelord: blå himmel [12]; polarisation [7]; *vardagsfysik [64]; #ljus [63];

1 http://science.howstuffworks.com/question39.htm

*

Ämnesområde
Sök efter
Grundskolan eller gymnasiet?
Nyckelord: (Enda villkor)
Definition: (Enda villkor)
 
 

Om du inte hittar svaret i databasen eller i

Sök i svenska Wikipedia:

- fråga gärna här.

 

 

Frågelådan innehåller 7624 frågor med svar.
Senaste ändringen i databasen gjordes 2022-05-21 17:33:39.

 

** Frågelådan är stängd för nya frågor tills vidare **


sök | söktips | Veckans fråga | alla 'Veckans fråga' | ämnen | dokumentation | ställ en fråga
till diskussionsfora

 

Creative Commons License

Denna sida från NRCF är licensierad under Creative Commons:
Erkännande-Ickekommersiell-Inga bearbetningar
.